Sei sulla pagina 1di 23

FÍSICA – MECÂNICA

DINÂMICA DA PARTÍCULA – Força, Energia, Momento Linear e Impulso.


SISTEMA DE PARTÍCULAS e CORPO RÍGIDO – Centro de Massa,
Rotação, Torque, Momeno Angular.
ESTÁTICA – Estática do ponto material, do corpo extenso e Fluidostática.
Pré- Vestibula r AUTORIA
da Universida de de Perna mbuco
MARCELO CORREIA
+++++++++++++++++++++++++++++++++++++++++++++++++++++++++++++++++++++++++++++++++++++++
Nesta parte estudaremos a DINÂMICA – parte da Mecânica que estuda os movimentos preocupando-se com as causas que os provocam ou os alteram.
Constata-se que em todas as provas de vestibulares é dada uma grande ênfase a Mecânica, portanto é muito importante o entendimento detalhado desta
parte da Física e além do mais a Mecânica constituirá uma base indispensável para o entendimento de outros ramos da Física. Durante o passar dos anos
compreendeu-se que a forma de passar no vestibular é muito simples, requer dedicação e uma fórmula mágica: estudar, estudar, estudar, ...
O professor de Física, MARCELO CORREIA.

DINÂMICA – PARTE 1 Devemos atentar para o seguinte:


A dinâmica é a parte da Mecânica que estuda os movimentos levando  A massa de um corpo é uma medida de sua inércia. Quanto maior a
em consideração as causas que os provocam ou os alteram. massa de um corpo maior a sua inércia, isto é, maior a sua resistência
a alterações do seu estado de repouso ou MRU;
LEIS DE NEWTON  Sabemos que um corpo em equilíbrio estático ou dinâmico não atua
r
As Leis de Newton constitui a base da Mecânica Clássica e são três: agente externo, isto é, FR = 0. Portanto quanto um corpo está em
1ª Lei de Newton – Princípio da Inércia repouso ou em MRU a força resultante que a tua sobre ele é nula, isto
O Princípio da inércia pode ser enunciado de diversas formas, então é o que chamamos de condição de equilíbrio de translação para
apresentaremos abaixo algumas formas de enunciar a 1ª Lei de Newton: uma partícula.
 FORÇA RESULTANTE é a soma vetorial de todas as forças que atua
no corpo.
TODO CORPO TEM A TENDÊNCIA DE PERMANECER NO SEU  A unidade de força no SI é: kg— m/s2 que recebe o nome especial de
ESTADO DE REPOUSO OU MRU AO MENOS QUE UM AGENTE newton – N.
EXTERNO ATUE SOBRE ELE.  1N é a força para que um corpo de 1kg adquira uma aceleração de
1m/s2.
TODO CORPO PERMANECERÁ EM REPOUSO OU EM MRU,
INDEFINIDAMENTE AO MENOS QUE UM AGENTE EXTERNO ATUE 3ª Lei de Newton – Princípio da Ação e Reação
SOBRE ELE. O princípio da ação e reação pode ser enunciado da forma seguinte:
r
QUALQUER VELOCIDADE, UMA VEZ TRANSMITIDA A UM CORPO, QUANTO UM CORPO “A” APLICA UMA FORÇA F NUM CORPO
r “B” O
SERÁ MANTIDA INDEFINIDAMENTE, DESDE QUE SOBRE O CORPO
NÃO ATUE CAUSAS PARA AQUISIÇÃO DE ACELERAÇÃO. CORPO “B” REAGE E APLICA NO CORPO “A” UMA FORÇA −F QUE TEM
MESMO MÓDULO, MESMA DIREÇÃO E SENTIDO OPOSTO.
Do Princípio da Inércia observamos que existem dois estados naturais
para o corpo, isto é, existem dois estados em que não é necessária a atuação de FORÇAS DE GRANDE IMPORTÂNCIA
r
agente externo para que o corpo esteja neles. Estes estados são: REPOUSO E Força Peso P
MRU (Movimento Retilíneo e Uniforme). Assim, podemos dizer que quando um A força peso é a força com que a Terra (ou outro corpo celeste
corpo se encontra em repouso ou em MRU ele está em equilíbrio. Portanto temos massivo) atrai os corpos que estão em suas proximidades para o seu centro.
dois tipos de equilíbrio: Observado de Fora da Terra Observado da Terra
 Repouso  Equilíbrio Estático;
 MRU  Equilíbrio Dinâmico. corpo m
r m Força peso
2ª Lei de Newton – Princípio Fundamental da Dinâmica
Observando que para um corpo não estar em repouso ou em MRU r P
−P r
deve sobre ele atuar um agente externo Newton definiu uma grandeza física que P
caracteriza quantitativamente a ação do agente externo e a esta grandeza deu o Reação da
nome de força. Assim, a 2ª Lei de Newton pode ser enunciada da forma seguinte: Força Peso
A FORÇA RESULTANTE QUE ATUA SOBRE UMA PARTÍCULA É Superfície
DIRETAMENTE PROPORCIONAL AO PRODUTO DE SUA MASSA (m) PELA terrestre
r
ACELERAÇÃO ADQUIRIDA POR ELA ( a ). Terra

Escrevendo o Princípio Fundamental da Dinâmica matematicamente No nosso caso, resolveremos a maioria das situações observando o
temos: fenômeno aqui da Terra, sendo assim a força peso que atua em uma partícula

r r Onde:
r
será uma força com direção vertical e sentido apontando para baixo. Sendo o

r
peso uma força deve obedecer a 2ª Lei de Newton, assim temos:

FR = m⋅ a
FR  força resultante
r r
m  massa do corpo
r
a  aceleração
P = m⋅ g g  aceleração da gravidade. Nas
proximidades da terra g = 100m/s2.

AUTORIA – PROF. MARCELO CORREIA 1 E-mail: marcelo.correia.fisica@bol.com.br


FÍSICA – MECÂNICA
r Força Elástica – Lei de Hooke
Reação Normal ou Normal N A Lei de Hooke trata da força elástica e pode ser enunciada da
A normal é uma força de reação provocada por um apoio, ou seja, é a seguinte forma:
força aplicada a um corpo pela superfície em que está apoiado. Não esqueça
que: AO APLICARMOS UMA FORÇA A UMA MOLA (CORPO ELÁSTICO) EM SEU
 A normal não é a reação da força peso; REGIME ELÁSTICO O MÓDULO DA FORÇA EXERCIDA PELO CORPO
 A ação que provoca a normal é aplicada no apoio, isto é, no corpo ELÁSTICO PARA RETORNAR AO SEU ESTADO NATURAL (LIVRE DE
em que o corpo está apoiado; FORÇAS) É DIRETAMENTE PROPORCIONAL A SUA DEFORMAÇÃO.
 Não há uma fórmula pronta para calcular a normal, portanto
devemos aplicar as Leis de Newton para encontrar a normal; A lei de Hooke escrita matematicamente é:
 Podemos, de forma coloquial, dizer que a normal é uma medida Fel  Força elástica
para o quanto sofre o apoio.
Vejamos algumas situações: F = K⋅ x
el
K  Constante elástica (depende da mola)
X  Deformação
r
N Veja abaixo:
Parede Mola livre de
Mesa
forças. Estado
r natural.
N xi x
r
Mola deformada. Fel
Aparece a força r
elástica. F
xf
r
N Nota: Um corpo está no regime elástico quando após a retirada das forças que o
deformam ele retorna ao seu estado inicial.
Teto r
Plano Força de Atrito Fat
Inclinado
A força de atrito é uma força de resistência a tendência do movimento.
A força de atrito tem origem em forças eletromagnéticas e por conta das
θ irregularidades das superfícies que tendem a escorar entre si.
r Consideramos dois tipos de forças de atrito:
N  Força de atrito dinâmica ou cinética  é a força de atrito que atua
sobre o corpo quando este está em movimento (v ≠ 0).
A experiência mostra que a intensidade da força de atrito dinâmica é
r diretamente proporcional a reação do apoio, isto é, é diretamente proporcional a
Tração ou Tensão T r
Chamamos de tração ou tensão a força que é transmitida através de normal N. Assim teremos:
Fat  Força atrito
Fat = µ d ⋅ N
um fio ou algo semelhante. Fio ideal é aquele que não tem massa, portanto não
tem inércia, e é inextensível (não se deforma). Não esqueça que: µd  Coeficiente de atrito dinâmico
 Para um mesmo fio ideal a tração nos seus extremos tem mesmo N  Reação do apoio – Normal
módulo e sentidos opostos.  Força de atrito estática  é a força de atrito que atua sobre o corpo
Veja algumas situações abaixo: quando este está em repouso (v = 0).
r A experiência mostra que a intensidade da força de atrito estática
F r
A r r B máxima é diretamente proporcional a normal N.
T −T
= µ ⋅N
Fat  Força atrito
F at(máx) e
µe  Coeficiente de atrito estático
N  Reação do apoio – Normal
É muito importante perceber que a expressão anterior nos fornece a
força de atrito estática máxima e esta não é necessariamente a força de atrito
r estática que pode estar atuando no corpo. Observamos que a força de atrito
T estática pode ter valores compreendidos entre zero e o valor máximo
A dependendo da força que solicita a movimentação do corpo (que permanece em
repouso). Assim podemos escrever: 0 ≤ F at ≤ F at(máx)
r
−T As forças de atrito estática e dinâmica não são iguais observe que há
dois coeficientes de atrito: o dinâmico (µd) e o estático (µe). Sabemos que o
B coeficiente de atrito estático é ligeiramente maior que o coeficiente de atrito
dinâmico, assim a força de atrito estática máxima é maior que a força de atrito
r dinâmica. Este é o motivo pelo qual é mais fácil manter um movimento do que
P
iniciar um movimento.
É muito importante perceber também que a força de atrito não tem
qualquer dependência com a área de contato entre as superfícies.

AUTORIA – PROF. MARCELO CORREIA 2 E-mail: marcelo.correia.fisica@bol.com.br


FÍSICA – MECÂNICA
r BRINQUEDINHO DE VESTIBULANDO!!!!!
Força de Resistência dos Fluidos R
Quando um corpo se movimenta imerso em um fluido (liquido ou gás); 1. (UAAM) Um pescador está sentado sobre o banco de uma canoa. A Terra
como o ar, por exemplo; aparece uma força de atrito causada pelo fluido ou força aplica-lhe uma força de atração gravitacional chamada peso. De acordo
de atrito viscoso ou simplesmente força de resistência. No caso do ar dizemos com a 3ª Lei de Newton, a reação dessa força atua sobre:
que é a força de resistência do ar. Observamos que experimentalmente a força (a) a canoa. (b) o banco da canoa.
r (c) a água. (d) a Terra.
de resistência R tem módulo diretamente proporcional a potência n do modulo (e) a canoa e a água e depende de canoa estar em repouso ou em movimento
da velocidade do fluido. Assim, podemos escrever:
2. (PUC–SP) No arremesso de peso, um atleta gira um corpo rapidamente e
R = k ⋅ vn depois o abandona. Se não houvesse a influência da Terra, a trajetória do
corpo após ser abandonado pelo atleta seria:
onde: (a) Circular. (b) Parabólica. (c) Curva qualquer.
 R  Intensidade da força de resistência;
(d) Retilínea. (e) Espira. (f)
 n  Constante que depende da ordem de grandeza da velocidade e
do tamanho do corpo. Para maioria dos casos: n = 1 ou n = 2; 3. Uma partícula sob a ação de várias forças cuja resultante é zero. Podemos
 K  Constante que depende da natureza do fluido (densidade, afirmar que a partícula:
temperatura) e depende da maior área de contado do corpo com o (a) Está em pouso.
fluido perpendicularmente a direção do movimento. (b) Está em movimento acelerado.
Nunca esqueça que a força de resistência sempre tem sentido (c) Está em movimento circular.
contrário ao movimento do corpo. (d) Está em movimento retilíneo uniforme.
(e) Pode estar em repouso ou em movimento retilíneo uniforme.
PLANO INCLINADO 4. (FUVEST–SP) Um veículo de massa 5,0 kg descreve uma trajetória
O plano inclinado é um dispositivo que aparece muito e, portanto retilínea e obedece à equação horária: S = 3— t2 + 3— t + 1, onde S é medido
vamos mostrar uma análise básica para se resolver problemas envolvendo o em metros e t em segundos. Qual o módulo da força resultante sobre o
plano inclinado. No caso que vamos mostrar não há atrito, no entanto se houver é veículo?
só incluir esta mais esta força.
y 5. (Fund. Carlos Chagas–SP) Para que um carrinho de massa m adquira uma
r certa aceleração de módulo a é necessário que a força resultante tenha
N módulo F. Qual é o módulo da força resultante para que um carrinho de
massa 2— m adquira uma aceleração de módulo 3— a?
r r (a) 1,5F (b) 2F (c) 3F (d) 5F (e) 6F
r Pt = Px
PN Pt = P ⋅ sen θ 6. (FE Itajubá–MG) Um corpo cujo peso é 4,0 N, sob a ação de uma força
θ constante, horizontal, de valor 3,0 N, descreve uma trajetória retilínea sobre
uma mesa horizontal, com uma velocidade constante de 2,0 m/s. Quanto
r x PN = P ⋅ cos θ vale o módulo da resultante das forças que atuam sobre o corpo?
P θ (a) 0,0N (b) 3,0N (c) 5,0
(d) Não se pode dizer coisa alguma a respeito do valor da resultante, uma vez
r que a situação descrita no problema é fisicamente impossível.
FORÇA EM TRAJETÓRIA CURVILÍNEA – FORÇA CENTRÍPETA FCP (e) Não se pode dizer coisa alguma a respeito do valor da resultante, uma vez
Sabemos da cinemática vetorial que uma partícula descrendo trajetória que, para isso, o problema não apresenta dados suficientes.
curvilínea é acelerada. Se uma partícula está se deslocando numa trajetória 7. (ITA–SP) Em seu livro “Viagem ao Céu”, Monteiro Lobato, pela boca de um
circular há a aceleração centrípeta que indica variação na direção da velocidade personagem, faz a seguinte afirmação: “Quando jogamos uma laranja para
vetorial da partícula. cima, ela sobe enquanto a força que produziu o movimento é maior do que
Agora que já estudamos a 2ª Lei de Newton sabemos que toda a força da gravidade. Quando esta se torna maior, a laranja cai”.
alteração de movimento (alteração de velocidade – aceleração) é causada por (Despreze a resistência do ar)
uma força. A força centrípeta é a força que provoca a aceleração centrípeta de (a) A afirmação é correta pois, de F = m— a, temos que a = 0 quando F = 0.
uma partícula e, portanto provoca variação na direção da velocidade vetorial da indicando que as duas forças se equilibram no ponto mais alto da trajetória.
partícula. Aplicando a 2ª Lei de Newton para a aceleração centrípeta temos: (b) A afirmação está errada porque a força exercida para elevar a laranja sendo
r r constante nunca será menor que a da gravidade.
FCP = m ⋅ a CP (c) A afirmação está errada porque, após ser abandonada no espaço, a única
força que age sobre a laranja é a da gravidade.
v2 (d) A afirmação está correta porque está de acordo com o Princípio da Ação e
Lembrando que: a CP = ou a CP = ω 2 ⋅ R , podemos calcular o Reação.
R (e) Não podemos tirar qualquer conclusão sobre a afirmação.
módulo da força centrípeta por:
8. (AEU–DF) As Leis de Newton da Dinâmica são verificadas:
v2 (a) Só para observadores em repouso.
FCP = m ⋅ ou FCP = m ⋅ ω 2 ⋅ R (b) Para quaisquer observadores.
2 (c) Só para observadores em movimento acelerado.
onde: (d) Para observadores parados ou com aceleração vetorial nula em relação a
 V  Módulo da velocidade vetorial da partícula; um sistema inercial.
 ω  velocidade angular da partícula; (e) Só para observadores em movimento uniforme.
 R  Raio da trajetória descrita pela partícula. 9. (PUC–SP) O sistema representado no desenho, de massa total 100 kg, é
Não esqueça que a força centrípeta tem mesma direção e mesmo r
sentido da aceleração centrípeta, isto é, aponta para o centro da trajetória. puxado para a direita por uma força F que o acelera uniformemente sobre
trilhos sem atrito. O dinamômetro D ligado à esfera E, de massa 10 kg, que
AUTORIA – PROF. MARCELO CORREIA 3 E-mail: marcelo.correia.fisica@bol.com.br
FÍSICA – MECÂNICA
pode deslizar sem atrito sobre a prancha horizontal, acusa uma força de 5 N 16. (F.C. Chagas – SP) Quatro blocos, M, N, P e Q, deslizam sobre uma
r r
durante a aceleração. A aceleração que F comunica ao sistema: superfície horizontal, empurrados por uma força F , conforme esquema
(a) Não pode ser determinado. r
(b) Vale 0,05m/s2.
F abaixo. A força de atrito entre os blocos e a superfície é desprezível, e a
E D
massa de cada bloco vale 3,0 kg. Sabendo-se que a aceleração escalar dos
(c) Vale 0,5m/s2. blocos vale 2,0 m/s2, a força do bloco M sobre o bloco N é, em newtons,
5 igual a:
(d) Vale m/s2. (a) 0 r
510
(b) 6 F M
5 P
(e) Vale m/s2. (c) 12 N Q
90 (d) 18
10. (ITA–SP) A velocidade de uma partícula, num determinado instante t, é nula (e) 24
em relação a um referencial inercial. Pode-se afirmar que o no instante t: 17. (UFES) Desprezando-se os atritos, a aceleração do bloco A será de:
(a) A resultante das forças que agem sobre a partícula é necessariamente nula. (a) 12,0 m/s2 r
3 kg
(b) A partícula se encontra em repouso, em relação a qualquer referencial (b) 9,8 m/s2 2 kg F = 24 N
inercial. (c) 4,8 m/s2 Parede
(c) A resultante das forças que agem sobre a partícula pode não ser nula. (d) 4,0 m/s2 A B
(d) A resultante das forças que agem sobre a partícula não pode ser nula. (e) 2,4 m/s 2

(e) Nenhuma das anteriores é verdadeira. 18. (Fatec – SP) A equação horária da velocidade de uma partícula em
11. (Mackenzie–SP) Um elevador começa a subir, a partir do andar térreo, com movimento retilíneo e de 3 kg de massa é v = 4 + 2— t, com unidades do
aceleração de 5 m/s2. O peso aparente de um homem de 60 kg no interior Sistema Internacional. A força resultante sobre a partícula tem módulo de:
do elevador, supondo g = 10 m/s2, é igual a: (a) 6 N (b) 2 N (c) 30 N (d) 3 N (e) 1,5 N
(a) 60N (b) 200N (c) 300N (d) 600N (e) 900N 19. (FEI – SP) Sabendo-se que a tração no fio que une os dois blocos vale
12. (ITA–SP) No teto de um elevador temos um corpo de peso 16 N preso a um
r
100N, qual é o valor do módulo da força F ? Não há atrito.
dinamômetro que acusa 20 N. A aceleração local da gravidade vale r
10 m/s2. A intensidade da aceleração do elevador é: F
(a) zero (b) 2,5m/s2 (c) 5,0m/s2 (d) 10,0m/s2 (e) n.d.a. Par 10 kg 5 kg
13. (ITA–SP) Em relação à situação da questão anterior, podemos afirmar que
o elevador está:
(a) subindo com velocidade constante.
(b) Em repouso. 20. (CESGRANRIO – RJ) Dois corpos de pesos respectivamente iguais a 20N e
(c) Subindo em movimento acelerado. 30N são mantidos em P
(d) Descendo em movimento acelerado. equilíbrio, como mostra
(e) Subindo em movimento acelerado ou descendo em movimento retardado. a figura. P representa
um dinamômetro de 20 N
14. (ITA–SP) No sistema esquematizado são desprezíveis o atrito, o momento 30 N
de inércia da roldana e a massa do fio que liga as massas m1 e m2. Sabe- massa desprezível.
se que m1 > m2 e que a aceleração da gravidade local é g. A tensão T no fio Qual a indicação do
e a aceleração a da massa m1 são, respectivamente, dadas por: dinamômetro?

(a) T =
2 ⋅ m1 ⋅ m 2 ⋅ g
; a=
(m1 − m2 ) ⋅ g (a) 50 N (b) 30 N (c) 20 N (d) 10 N (e) zero
m1 + m 2 m1 + m 2 21. (UNIMEP–SP) Um corpo A de massa 1600 gramas está unido por um fio a
(b) T =
m1 ⋅ m 2 ⋅ g
; a=
(m1 − m2 ) ⋅ g um outro corpo B de massa 400 gramas, numa região em que g = 10 m/s2.
No instante inicial, o corpo A tinha uma velocidade de 5 m/s e se movia para
m1 + m 2 m1 + m 2 direita, conforme o esquema.
(c) T = (m1 − m2 ) ⋅ g ; a =
(m1 − m2 ) ⋅ g Desprezando-se os atritos, após
v0 = 5m/s
5s, o módulo e o sentido da A
m1 + m 2 velocidade de A serão:
(d) T = (m1 − m2 ) ⋅ g ; a =
(m1 − m2 ) ⋅ g (a) v = 5m/s; da esquerda para direita.
m1
m1 (b) v = 0m/s; da esquerda para direita.
(c) v = 0 m/s; da direita para esquerda. B
(e) T = (m1 + m2 ) ⋅ g ; a =
(m1 + m2 ) ⋅ g m2 (d) v = 5m/s; da direita para esquerda.
m1 (e) v = 2m/s; da esquerda para direita.
15. (UFPI) A figura mostra dois blocos sobre uma mesa lisa plana e horizontal. 22. (FATEC–SP) No sistema esquematizado da figura, os blocos A e B têm
As massas dos blocos são m1 = 2 kg e m2 = 8 kg. Ao sistema é aplicada massas respectivamente iguais a mA e mB, o fio é ideal e não há atritos.
uma força F, horizontal, de intensidade 40 N. A intensidade da força que o Sendo g a aceleração da gravidade e T a tração no fio quando o sistema
bloco m1 exerce sobre o bloco m2 é: está em movimento, podemos afirmar que:
(a) 4N (a) T = mA — g B
(b) 8N r (b) T = mB — g
m2 (c) T > mB — g
(c) 24 N F m1
(d) 32 N (d) T = (mA + mB) — g
(e) T < mA — g A
(e) 40 N

AUTORIA – PROF. MARCELO CORREIA 4 E-mail: marcelo.correia.fisica@bol.com.br


FÍSICA – MECÂNICA
23. (PUC–SP) O esquema representa dois corpos A e B de massas (a) zero.
respectivamente igual a 8,0 kg e 2,0 kg, ligados por um fio inextensível e de (b) Menor que 30º.
massa desprezível. No instante t = 0 os corpos estão em repouso na (c) Menor que 45º.
posição indicada no esquema. Nesse instante abandona-se o sistema, que (d) Diferente de 90º.
assume movimento devido à tração exercida por B. Despreze as forças de (e) O bloco não entra em movimento qualquer que será θ.
atrito e suponha que a aceleração da gravidade tem intensidade 10 m/s2. O
28. (ITA – SP) rUm vagão desloca-se horizontalmente, em linha reta, com
tempo que A leva para ir de M até N é:
(a) 1,0 s aceleração a constante. Um pêndulo simples está suspenso do teto do
A vagão, sem oscilar e formando ângulo θ com a vertical. Sendo g a
(b) 2 s aceleração da gravidade e m a massa do pêndulo. O módulo da tensão T no
(c) 2,0 s fio do pêndulo é:
(d) 5 s (a) T = m ⋅ g ⋅ cosθ (b) T = m ⋅ a ⋅ senθ
(e) 3,0 s 5m (c) T = m ⋅ a + g 2 2 (d) T = m ⋅ (g ⋅ cosθ − a ⋅ senθ )
M N B (e) T = m ⋅ (g ⋅ senθ + a ⋅ cosθ )
29. (UFGO) Um bloco desliza sobre um plano horizontal sem atrito com
r
1,0 m velocidade constante v 0 . Em seguida, ele sobre uma rampa de inclinação
solo
θ, também sem atrito, até parar no ponto C da figura. A distância BC
24. (Fund. Carlos Chagas – SP) A figura mostra um sistema de roldanas percorrida ao longo da rampa é:
C
sustentando uma lâmpada. r
Os atritos e as massas das v0
roldanas e das cordas são B θ
desprezíveis. A lâmpada L,
2⋅ v0
2 2 2
cujo peso é P newtons, é v0 v0
equilibrada pelo peso X, (a) (b) (c)
cujo valor em newtons é: 2 ⋅ g ⋅ tgθ 2 ⋅ g ⋅ cosθ g ⋅ senθ
(a) P/2 v0
2 2
v0
(b) P (d) (e)
(c) 2— P
g ⋅ tgθ 2 ⋅ g ⋅ senθ
(d) 4— P L
30. (Mack – SP) Os corpos A (mA = 2,0 kg) e B (mB = 4,0 kg) da figura abaixo
r
(e) 6— P X sobem a rampa com movimento uniforme, devido à ação da força F ,
paralela ao plano inclinado. Despreze os atritos e adote g = 10 m/s2. A
25. (FEI – SP) No sistema da intensidade da força que A exerce em B é de:
figura ao lado, o fio e as (a) 2,0 N
polia são ideais. Qual a
A
(b) 3,0 N B
aA (c) 20 N r A
relação entre as
(d) 30 N F
aB 30º
(e) 40 N
acelerações adquiridas
pelos corpos A e B, 31. (ITA – SP) O plano inclinado da figura tem massa M e sobre ele se apóia um
sabendo que as massa objeto de massa m. O ângulo de inclinação é α e não há atrito nem entre o
obedecem à relação plano inclinado e o objeto, nem entre o plano inclinado e o apoio horizontal.
r
mA 1 B Aplica-se uma força F horizontalmente ao plano inclinado e constata-se
= ? Despreze o
que o sistema todo se move horizontalmente sem que o objeto deslize em
mB 4
relação ao plano inclinado. Podemos afirmar que, sendo g a aceleração da
atrito. gravidade local:
26. (CESCEA – SP) Dois corpos A e B, de massas respectivamente iguais a (a) F = m—g r
2kg e 4kg, estão encostados um no outro e podem se deslocar sem atrito m
r (b) F = (M + m)— g F
sobre um plano horizontal. Sobre o corpo A é aplicada a força FA de (c) F tem que ser infinitamente grande M
r (d) F = (M + m)— g—tgα α
módulo 12N e sobre o corpo B é aplicada a força FB de módulo 6N, (e) F = Mg—senα
conforme a figura. A aceleração do conjunto vale, aproximadamente: 32. (PUC – SP) Dois blocos A e B, de pesos respectivamente iguais a 30N e
(a) 3,0 m/s2 r 70N, apóiam-se sobre uma mesa horizontal, ligados por um fio ideal. O
(b) 1,41 m/s2 r A F
45º A coeficiente de atrito entre os blocos e a mesa é 0,4 e a aceleração da
(c) 1,0 m/s2 FB B
r
gravidade é g = 10 m/s2. Aplicando-se ao bloco A uma força horizontal F
(d) 0,41 m/s2 de intensidade 50 N, a aceleração comunicada ao sistema é:
(e) zero (a) 5 m/s2
27. (PUC – SP) Um bloco apoiado sobre uma superfície horizontal sem atrito (b) 4 m/s2 r
está inicialmente em repouso. A seguir, aplica-se ao bloco uma força de (c) 3 m/s 2 F
intensidade igual à metade de seu peso, numa direção que forma um ângulo B A
(d) 2 m/s2
θ com a horizontal. O valor de θ para que o bloco entre em movimento é
(e) 1 m/s2
necessariamente:
AUTORIA – PROF. MARCELO CORREIA 5 E-mail: marcelo.correia.fisica@bol.com.br
FÍSICA – MECÂNICA
33. (PUC – SP) Para o caso da questão anterior, a tração no fio vale: 41. (FEI – SP) No sistema da figura, o corpo A tem peso 200 N, as molas M1
(a) 50 N (b) 35 N (c) 25 N (d) 15 N (e) 10 N e M2 possuem constantes
34. (Fund. Carlos Chagas – SP) Um corpo de massa igual a 4,0 kg desloca-se elásticas k1 = 103 N/m e k2 = 2—
sobre uma superfície plana horizontal, ao longo de uma linha reta, com 103 N/m. As molas e as polias
velocidade escalar constante e igual a 2,0 m/s. O corpo se move sob a ação são ideais. As deformações M1
de uma força constante cuja direção é paralela à trajetória do corpo e cuja produzidas nas molas M1 e M2
intensidade é 3,0 newtons. Podemos afirmar que o módulo da força de valem, respectivamente:
atrito entre o corpo e a superfície é: (a) 10 cm e 5 cm
(a) 3,0 N (b) 5,0 N (c) 6,0 N (d) 8,0 N (e) 11,0 N (b) 20 cm e 0 cm
35. (FEI – SP) Lança-se um corpo num plano horizontal com velocidade (c) 20 cm e 10 cm
v0=10m/s. O corpo desloca-se sobre o plano e pára após 10s. Dado (d) 10 cm e 10 cm M2
g=10m/s2, calcule o coeficiente de atrito entre o bloco e a superfície. (e) N.d.a.
A
(a) 0,1 (b) 0,2 (c) 0,3 (d) 0,4 (e) 0,5
36. (PUC – SP) No sistema da figura, os corpos A e B têm massas mA = 6 kg e
mB = 4 kg, respectivamente. O fio que os une e a polia são ideais. A
resistência do ar é desprezível, a aceleração local da gravidade é g=10m/s2,
e o coeficiente de atrito entre o corpo A e o plano horizontal é µ. Quando o
sistema é abandonado em repouso na posição indicada, os blocos adquirem 42. (FATEC – SP) A figura indica um corpo A de 4 kg preso na extremidade de
uma mola, de constante elástica
aceleração de 1m/s2. Nessas condições, podemos afirmar que o valor de µ
100 N/m, apoiado numa mesa.
é: A Nestas condições a mola
(a) 0,10 experimenta um aumento de
(b) 0,25 comprimento de 10 cm.
(c) 0,30 Considerando-se g = 10m/s2,
B
(d) 0,50 podemos afirmar que a mesa
(e) 0,75 exercerá sobre o corpo A uma
37. (FATEC – SP) Uma caixa desliza ao longo de um plano inclinado com atrito força de intensidade:
e inclinação θ em relação a horizontal. Ao ser aumentado o ângulo θ, a força (a) 40 N
A
de atrito: (b) 30 N
(a) Não se altera. (c) 20 N mesa
(b) Aumenta de intensidade. (d) 10 N
(c) Muda de sentido mas não de intensidade. (e) 00 N
(d) Diminui de intensidade. 43. (FEI – SP) Os corpos A e B representados na figura possuem,
(e) Nenhuma das anteriores. respectivamente, massas mA = 2,0 kg e mB = 4,0 kg. A mola é ideal e tem
r
38. (FM ABC – SP) Um bloco de metal é colocado sobre uma mesa horizontal constante elástica k = 50 N/m. Despreze os atritos. Aplicando a força F
que se vai inclinando gradualmente. Quando a mesa forma com a horizontal constante e horizontal, verifica-se que a mola experimenta deformação de
o ângulo θ da figura, o bloco fica na iminência de deslizar. O coeficiente de 20cm. Calcule as intensidades:
atrito, entre o bloco e a mesa, vale: I. Da aceleração do conjunto;
(a) 0,20 r r
(b) 0,30 II. Da força F . F
A B
(c) 0,40
(d) 0,10 15 cm
θ
(e) 0,70
50 cm
44. (VUNESP – SP) Uma pedra de massa m = 0,20 kg gira, presa a um fio,
39. (FATEC – SP) Com pára-quedas aberto, um soldado salta de um helicóptero descrevendo uma circunferência horizontal de raio R = 20 cm, enquanto
em grande altura acima de uma planície. Sobre o sistema formado pelo perfaz 2,0 rotações por segundo. Tentando aumentar a velocidade
pára-quedas e pelo homem, podemos afirmar que: angular, vermos que o fio se rompe. Calcule a tração máxima que o fio
(a) a velocidade cresce uniformemente com aceleração inferior a g. suporta (g = 10 m/s2).
(b) a velocidade de chegada ao solo depende da altura inicial. (a) 10 N (b) 6,0 N (c) 6,3 N (d) 2,0 N (e) 6,6 N
(c) A velocidade de chegada ao solo depende da duração do processo. 45. (CESCEM – SP) Uma esfera de massa 0,50 kg oscila no vácuo suspensa
(d) À medida que a velocidade se eleva, aumenta a força resultante que as por um fio de 1 m de comprimento. Ao passar pela parte mais baixa da
cordas exercem no homem.
(e) Nenhuma das anteriores. trajetória, ela tem velocidade de 10 m/s. Aceleração da gravidade
g=10m/s2. O valor da intensidade da força de tração no fio, na parte mais
40. (ITA – SP) Numa região em que g = 10m/s2, um corpo de massa m = 8,0
baixa da trajetória, é um valor expresso em newtons, igual a:
gramas cai na água, atingindo após alguns segundos uma velocidade
praticamente constante de 5,0m/s. Sabe-se que, neste caso, a força de (a) 10,0 (b) 8,0 (c) 7,5 (d) 5,0 (e) zero
resistência exercida pela água é dada pro Fr = k— v, onde v é a velocidade 46. (OSEC – SP) Um motociclista descreve uma circunferência vertical num
do corpo. Desprezando o empuxo da água, podemos afirmar que a “globo da morte” de raio 4 m. Que força é exercida sobre o globo no ponto
constante k é igual a: mais alto da trajetória se a velocidade da moto é ali de 12 m/s? A massa
(a) 16 N— s/m (b) 1,6— 102 kg/s (c) 1,6— 103 kgf/s total (motociclista + moto) é de 150 kg.
(d) 1,6— 10 N— s/m
–3 (e) N.d.a. (a) 1500 N (b) 2400 N (c) 3900 N (d) 5400 N (e) 6900 N
AUTORIA – PROF. MARCELO CORREIA 6 E-mail: marcelo.correia.fisica@bol.com.br
FÍSICA – MECÂNICA
47. (UFPA) Um pequeno corpo de massa m está preso à extremidade de um BRINCANDO COM A COVEST (UFPE – UFRPE)
acorda de comprimento L e gira com velocidade angular ω em uma 56. A figura abaixo representa uma polia sem
circunferência vertical. A tração na corda, quando o corpo está no ponto massa e sem atrito. Os corpos de massa
mais alto da trajetória é, dada por: mA = 4 kg e mB = 1 kg estão presos a uma

( ) (b) m  ω 2 L − g  corda inextensível e de massa desprezível.


(a) m ω 2 L + g  4  (c) m ω 2L − g Qual o módulo da aceleração do corpo de
  massa mA, em m/s2?
m (ω L − g )
2 mA
(d) (e) m ω 2L + g mB
48. (UnB – DF) Um certo trecho de uma montanha-russa é aproximadamente 57. No plano inclinado da figura
um arco de circunferência de raio R. Os ocupantes de um carrinho, ao seguinte, o bloco de massa M
passar por este trecho, sentem uma sensação de aumento de peso. Avaliam desce com aceleração a = 2
que, no máximo, o seu peso foi triplicado. Desprezando os efeitos de atritos, m/s2, puxando o bloco de
os ocupantes concluirão que a velocidade máxima atingida foi de: massa m. Sabendo-se que M
(a) 3gR (b) 3 gR (c) 2 gR (d) 2gR (e) gR não há atrito de qualquer
espécie, qual o valor da razão 30º m
49. (EE MAUA – SP) Numa estrada existe uma curva circular plana de raio
150m. O coeficiente de atrito lateral entre o pneu e a estrada é 0,15. M ? g = 10 m/s2.
m
Determine a maior velocidade com que o carro pode percorrer a curva sem
derrapar. (g = 10 m/s2). 58. Uma pessoa puxa um bloco de
massa 0,2 kg com auxílio de uma
50. (OSEC – SP) Um toca-discos tem o prato na posição horizontal e realiza 3
mola de constante elástica igual 20
revoluções em π segundos. Colocando-se uma pequena moeda sobre o
N/m, conforme a figura. Se o
prato, ela deslizará se estiver a mais de 10 cm do centro. Então, o
coeficiente de atrito estático entre o
coeficiente de atrito estático entre a moeda e o prato é de:
bloco e a superfície horizontal é
(a) 0,12 (b) 0,24 (c) 0,36 (d) 0,48 (e) n.d.a. 0,5, qual o alongamento máximo
51. (FEI – SP) Um esfera gira com velocidade 1 m/s, descrevendo uma da mola, em cm, que ainda
trajetória circular, horizontal, de raio R = 10 cm. Estando a esfera suspensa mantém o bloco em repouso?
por meio de um fio. Qual o ângulo que este fio forma com a vertical? 59. Uma gota de chuva, de massa igual a 0,05 g, cai verticalmente com
Adote g = 10 m/s2. velocidade constante. Qual a força de resistência do ar, atuando sobre a
52. (PUC – SP) O raio de uma curva ferroviária é de 400 metros e um trem gota, em unidades de 10–5N?
deve percorrê-la com velocidade de 72 km/h. De quanto deve estar elevado 60. No sistema mostrado na figura,
o trilho externo para reduzir a um mínimo a força para fora sobre ele? A o bloco tem massa igual a 5,0
distância entre os trilhos é de 1,2 metros e g = 10 m/s2. kg. A constante elástica da
(a) 0,20m (b) 0,12m (c) 0,15m (d) 0,18m mola, 2,0 N/m. Considere que o
53. (Mackenzie – SP) Admitamos que você esteja apoiado, em pé, sobre o fio, a mola e a roldana são
fundo de um cilindro de 4 m de raio, que gira em torno do seu eixo vertical. ideais. Na situação de equilíbrio,
Admitindo g = 10 m/s2 e o coeficiente de atrito entre a sua roupa e a qual a deformação da mola, em
superfície do cilindro igual a 0,4 a mínima velocidade tangencial que o cm?
cilindro deve ter para que, retirado o fundo do mesmo, você fique “preso” à 61. A figura mostra dois blocos em repouso. O coeficiente de atrito estático
parede dele, é: entre o bloco B, de massa 30kg,
(a) 10 m/s (b) 8 m/s (c) 9 m/s (d) 11 m/s A
e a superfície de apoio é 0,6.
(e) é necessário conhecer a sua massa, pois sem ela nada se pode afirmar. considere que a polia e o fio são
54. Um homem de massa mH = 80 kg está sobre uma balança de molas, a qual ideais. Qual o maior valor, em
está fixa no piso de um elevador. A massa do elevador juntamente com a kg, da massa do bloco A para B
balança é mE = 520 kg. O conjunto está inicialmente em repouso. A partir de que o sistema permaneça em
determinado instante, aplica-se ao teto do elevador, através do cabo que repouso?
sustentação, uma força vertical para cima de modo que o elevador começa 62. Um bloco de 6,0kg sobe o plano inclinado na figura, sob a ação de uma
a subir com movimento acelerado,cuja aceleração tem módulo a = 2 m/s2. força externa paralela ao plano. O coeficiente de atrito entre o plano e o
I. Calcule a intensidade da força aplicada ao teto do elevador; r
II. Calcule a marcação da balança. F
55. O sistema esquematizado na figura está inicialmente em repouso. O fio e a
polia são ideais, g = 10 m/s2, a massa de A é 4,0 kg e a de B é 16 kg.
Existe atrito entre A e B e entre B e a superfície de apoio, sendo o
coeficiente de atrito dinâmico igual a 0,20 em ambos os casos.
30º
A partir de determinado instante,
aplica-se ao bloco B uma força A r 3
r bloco é µ = . Qual será o valor da força externa F, em newtons, para
horizontal F , como mostra B
F
r 2
figura. Calcule o módulo de F que o bloco esteja em movimento uniforme?
nos seguintes casos: 63. Um objeto de 2,0kg descreve uma trajetória que obedece à equação horária
I. Os blocos passam a mover-se com velocidade constante; S = 5,0 + 3,0— t + 7— t2, onde S é medido em metros e t em segundos. O
II. Os blocos passam a mover-se com aceleração constante de módulo da força resultante que está atuando sobre o objeto é, em N.
módulo a = 3,0 m/s2. (a) 10 (b) 17 (c) 19 (d) 28 (e) 35
AUTORIA – PROF. MARCELO CORREIA 7 E-mail: marcelo.correia.fisica@bol.com.br
FÍSICA – MECÂNICA
64. Um físico, atendendo à sua esposa, tenta mudar a localização da sua mostra a figura. Os valores em N, das forças resultantes que atuam sobre os
geladeira empurrando-a horizontalmente sobre o chão, mas não consegue blocos A e B são respectivamente:
movê-la. Pensando sobre o assunto, ele imagina como sua vida seria mais (a) 40 e 50
fácil num planeta de gravidade menor que a da Terra. Considerando que a (b) 45 e 45 r B
força que o físico faz sobre a geladeira vale 1200N, a massa da geladeira é (c) 90 e 90 F A
300kg, e o coeficiente de atrito estático entre a geladeira e o chão é ½, (d) 20 e 70
indique entre os planetas abaixo aquele com maior aceleração da gravidade, (e) 30 e 60
g, no qual ele ainda conseguiria mover a geladeira.
69. Um pequeno bloco de 0,50 kg desliza sobre um plano horizontal sem atrito,
(a) Plutão, g = 0,3 m/s2 (b) Marte, g = 3,7 m/s2 sendo puxado por uma força constante F = 10,0 N aplicada a um fio
(c) Urano, g = 7,8 m/s 2 (d) Vênus, g = 8,6 m/s2 inextensível que passa por uma roldana, conforme a figura. Qual a
(e) Saturno, g = 9,0 m/s2 aceleração do bloco, em m/s2, na direção paralela ao plano, no instante em
65. Um corpo de massa igual a 10,0 kg é suspenso em uma balança de mola que ele perde o contanto com o plano? Despreze as massas do fio e da
presa ao teto de um elevador. Calcule a diferença, em newtons, entre as roldana, bem como o atrito no eixo da roldana.
leituras na balança quando o elevador está subindo ou descendo, sabendo (a) 12,4
que nos dois casos a aceleração tem módulo a = 0,5 m/s2. (b) 14,5
66. Uma partícula F (c) 15,2
inicialmente em repouso (d) 17,3 F
é submetida à ação da F0 (e) 18,1
força mostrada no 70. Um bloco de massa 1,5 kg é solto, a partir do repouso, do topo de um plano
gráfico abaixo. Indique inclinado de 5,0 m de altura,
2t0
qual dos gráficos melhor
t0 t conforme a figura. O tempo
representa a variação
– F0 gasto pelo bloco para descer até
da posição da partícula a base do plano é igual a 2,0 s.
em função do tempo. H = 5,0 m
Qual o comprimento do plano
inclinado, em metros? Despreze
(a) (b) o atrito entre o bloco e o plano.
x x
BRINCANDO COM A UPE
71. Um bloco de massa m = 5 kg esta subindo a rampa inclinada de 30º com a
t
r O atrito vale 40% do peso do
horizontal, mantendo a velocidade constante.
bloco. A força F tem módulo igual a: F
t0 2t0 t t0 2t0 (a) 25 N
(b) 30 N
(c) 35 N
(d) 40 N
(e) 45 N 30º
(c) x (d) x
72. O caminhão altera a velocidade altera a velocidade de 54 para 90 km/h num
tempo de 10 segundos. O caixão de 1000 kg não desliza sobre a
carroceria. Qual a força de atrito, em kgf, na superfície de contato entre o
t t caminhão e o caixote?
t0 2t0 t0 (a) 100 (b) 80 (c) 40 (d) 60 (e) 20
2t0
73. No dimensionamento de um novo automóvel, pesando 1600kgf,
estabeleceu-se que o rebaixamento máximo das molas com a carga máxima
de 5 pessoas de 80kg é de 5cm. Qual deve ser a constante elástica (em
N/m) comum às quatro molas?
(e)
x (a) 50000 (b) 40000 (c) 30000 (d) 20000 (e) 10000
74. Uma pessoa comprou uma balança de chão e, ao chegar em casa, ansiosa
F0 para controlar o peso, resolve testá-la ainda no elevador. Ela conclui que a
balança estava com defeito ao notar um aumento de seu peso.
2t0 Considerando essas informações, identifique a opção correta.
t0 t (a) O aumento da indicação da balança pode ocorrer se o elevador está
– F0 subindo com velocidade constante.
(b) O aumento da indicação da balança pode ocorrer se o elevador está
descendo com velocidade constante.
(c) O aumento da indicação da balança pode ocorrer se o elevador está
67. Um homem sobe numa balança no interior de um elevador. Com o elevador subindo com aceleração constante.
parado a indicação da balança é de 60kg. Se o elevador estiver subindo (d) O aumento da indicação da balança pode ocorrer se o elevador está
com aceleração de 2 m/s2, qual será a indicação da balança. (g = 10 m/s2) descendo com aceleração constante.
(a) 48 kg (b) 60 kg (c) 72 kg (d) 84 kg (e) 96 kg (e) A balança está necessariamente com defeito e deve ser trocada em respeito
68. Dois blocos A e B de massas respectivamente iguais a 5 kg e 10 kg está aos direitos do consumidor.
inicialmente em repouso, encostados um no outro, sobre uma mesa
horizontal sem atrito. Aplicando-se uma força horizontal F = 90 N, como
AUTORIA – PROF. MARCELO CORREIA 8 E-mail: marcelo.correia.fisica@bol.com.br
FÍSICA – MECÂNICA
75. Um corpo de massa m está em repouso sobre um plano inclinado que faz DINÂMICA – PARTE 2
um ângulo de 30º com a horizontal, conforme mostra a figura. Em relação a Nesta segunda parte do estudo da dinâmica vamos estudar grandezas
essa situação, analise as seguintes proposições. físicas muito importantes: Trabalho e potência, energia. Vamos discutir um
I II princípio de conservação muito importante: O princípio de conservação da
A força normal que o energia mecânica.
plano inclinado exerce
sobre o corpo de
a TRABALHO τ
Em Física a palavra trabalho não tem exatamente o mesmo significado
0 0 massa m está na 30º
que usamos cotidianamente. Em Física o trabalho é realizado quando uma força
direção vertical, no
“ajuda” ou “atrapalha” um deslocamento.
sentido de baixo para b
Vamos considerar um corpo de massa m que efetue um deslocamento
cima.
para direita com módulo d, onde no mesmo atua uma força constante com
1 1 A força de atrito é maior do que m— g. r
módulo F, como mostra a figura a seguir:
2 2 A força peso é perpendicular ao plano inclinado. F r
3 3 A força de atrito é igual a m— g— sen30º. θ d
4 4 A força normal é igual a m— g— cos(a/b).
76. Uma menina está no carro de uma montanha-russa, que faz uma volta d
circular completa na vertical. No topo da trajetória, a força normal exercida
pela cadeira sobre a menina é igual a duas vezes o peso da menina, 2mg.
r
Nestas condições, se a força
r F que atua no corpo durante um
No ponto mais baixo da trajetória, a força normal exercida pela cadeira
sobre a menina é: deslocamento d for constante podemos calcular o trabalho efetuando o produto

τ
(a) Menor do que o peso da menina. escalar entre o vetor força e o vetor deslocamento, assim temos:
r r
(b) Igual ao peso da menina.
(c) Igual à força normal no topo da trajetória. = F•d
(d) Igual a quatro vezes o peso da menina. Mas, como já sabemos da álgebra vetorial um produto escalar entre

τ = F ⋅ d ⋅ cosθ
(e) Igual a oito vezes o peso da menina. dois vetores pode ser calculado por:
77. Em relação ao conceito e ao tipo de força.
I II Se analisarmos a expressão anterior podemos perceber facilmente
0 0 As forças de ação e reação sempre atuam em corpos distintos. que:
r
1 1 A força elástica é proporcional à deformação da mola. • 0º ≤ θ < 90º o trabalho realizado pela força F é positivo, neste caso
2 2 A força normal é uma força de reação ao peso. chamamos este trabalho de trabalho motor e isto significa que a força
3 3 Força é uma grandeza vetorial. “ajuda” o deslocamento. Neste caso a força transfere energia para o
4 4 Uma força sempre causa mudança no valor da velocidade. corpo;
r
• θ = 90º o trabalho realizado pela força F é nulo, neste caso a força
nem “ajuda” nem “atrapalha” o deslocamento do corpo;
ANOTAÇÕES / OBSERVAÇÕES r
• 90º < θ ≤ 180º o trabalho realizado pela força F é negativo, neste
caso chamamos este trabalho de trabalho resistente e isto significa
que a força “atrapalha” o deslocamento. Neste caso a força retira
energia do corpo.
Você percebe que o trabalho é uma grandeza que indica a
transformação de energia. Quando uma força realiza trabalho ela pode
transformar energia de uma forma em outra e o sistema sobre o qual o trabalho
está sendo realizado pode está recebendo energia ou pode está sendo retirada
energia do sistema.
A unidade de trabalho no SI é o N— m (Newton vezes metro) que
recebe o nome especial de J (joule), assim: 1 N— m = 1 J.
Podemos destacar três situações tem grande freqüência de aparição
nas questões a serem resolvidas:
• Quando a força tem mesma direção e mesmo sentido do

τ = F⋅d
deslocamento: Neste caso podemos calcular o trabalho pela

expressão: ;
• Quando a força tem mesma direção e sentido oposto ao

τ = −F ⋅ d
deslocamento: Neste caso podemos calcular o trabalho pela

expressão:

τ =0
• Quando a força é perpendicular ao deslocamento: Neste caso não

a realização de trabalho, isto é: .


É importante lembrar que se num corpo atuar diversas forças cada
força realiza o seu trabalho independente das demais forças e o trabalho
realizado pela força resultante é a soma algébrica dos trabalhos realizados por
todas as forças individualmente.
AUTORIA – PROF. MARCELO CORREIA 9 E-mail: marcelo.correia.fisica@bol.com.br
FÍSICA – MECÂNICA
Trabalho Realizado pela Força Peso Onde θ é o ângulo formado entre o vetor força e o vetor velocidade do
Aplicando a definição de trabalho realizado por uma força mostra corpo. Para o caso especial em que a força e a velocidade têm mesma direção e
Pot = F ⋅ v .
anteriormente, podemos calcular o trabalho realizado pela força peso. Assim
podemos observar duas situações em que a força peso realiza trabalho: mesmo sentido podemos escrever:
• Na descida de um corpo de uma determinada altura “h”: Neste A unidade de potência no sistema internacional (SI) é o J/s (joule por

τ = P ⋅h τ = m ⋅ g ⋅h
caso constatamos que o trabalho realizado pode ser calculado pela segundo) que recebe o nome especial de W (watt), assim temos: 1 J/s = 1 W.
Outras unidades de potência usadas são o: HP (horse-power) e o CV (cavalo-
expressão: ou ;
vapor). As relações são: 1 CV = 735 W; 1 HP = 745,7 W ≈ 746 W.
• Na subida de um corpo a uma determinada altura “h”: Neste caso

τ = −P ⋅ h τ = −m ⋅ g ⋅ h
constatamos que o trabalho realizado pode ser calculado pela Gráfico da Potência em Função do Tempo
No gráfico da potência em função do tempo a área limitada pelo
expressão: ou . gráfico e o eixo dos tempos é numericamente igual ao trabalho realizado no
intervalo de tempo considerado.
Gráfico Fx S
No gráfico da força F em função da posição S a área limitada pelo
gráfico e o eixo das posições é numericamente igual ao trabalho realizado pela
RENDIMENTO η
força no deslocamento d considerado. Imagine uma máquina qualquer que deve realizar determinado
F trabalho. Digamos que seja fornecida certa energia para que esta máquina
realize tal trabalho, no entanto, parte desta energia é desperdiçada por forças
dissipativas, tal como a força de atrito.
O rendimento é uma grandeza física que mede a eficiência de uma
máquina, isto é, o rendimento mede quanta da energia fornecida a uma máquina
τ =A é usada por ela para realizar um certo trabalho útil e, conseqüentemente mede
quanto ela dissipa usando para realizar trabalho em forma não útil. Assim, quanto
0 S
d maior for o rendimento de uma máquina mais eficiente ele é.
Podemos então destacar três tipos de trabalho ou potências:
• Trabalho útil • Potência útil
• Trabalho dissipado • Potência dissipada
τ = F ⋅ d ⋅ cosθ , que usamos para calcular o trabalho não poderá ser usada
Quando uma força variar com a posição a expressão:
• Trabalho total • Potência total

τ =τ +τ
pois esta expressão só é válida para o caso em que a força é constante. Quando De forma que podemos escrever:
a força variar com a posição podemos calcular o trabalho recorrendo ao gráfico
da força em função da posição como descrito acima. T u d e TP = Pu + Pd

τ
Assim o rendimento pode ser dado por:
Trabalho Realizado pela Força Elástica
Pu
η=
τ η=
A força elástica é uma força que varia com a posição. Portanto par u
calcularmos o trabalho realizado pela força elástica temos que recorrer ao gráfico
da força elástica em função da posição, que neste caso é a deformação “x” da T
ou
PT
mola. Fazendo isto obtemos a seguinte expressão: Note que o rendimento é uma grandeza adimensional (que não tem

τ = ± k ⋅2x
2 unidade) já que é uma relação entre mesmas grandezas. O rendimento é,
sempre, um valor entre zero e um, isto é: 0 ≤ η≤
1 . Uma máquina que tenha
rendimento igual a 1 (um) é uma máquina ideal.
Onde “k” é a constante elástica (ou constante de força) da mola, “x” é Podemos expressão o rendimento em percentual. Para obter o
rendimento percentual basta multiplicar o rendimento por 100. Assim temos:
η % = 100 ⋅η
a deformação da mola. O sinal mais o menos é determinado observando-se se a
força elástica se opõe (caso em que usamos menos) ou favorece o deslocamento
(caso em que usamos mais).

POTÊNCIA ( Pot ) BRINQUEDINHO DE VESTIBULANDO!!!!!


Imagine duas pessoas que realizam o mesmo trabalho. Porém, uma 78. Um ponto material de massa 6kg tem velocidade de 8m/s quando sobre ele
delas realiza o trabalho num intervalo de tempo menor que a outra. Então, a de passa a agir uma força de intensidade 30N na mesma direção e sentido do
se pensar que a pessoa mais rápida é mais eficiente do que a que realizou o movimento, durante 4s. Determine:
mesmo trabalho num intervalo de tempo maior. Para diferenciar a com o trabalho I. O deslocamento durante esses 4s;
está sendo realizado em relação ao tempo usamos a grandeza física potência. II. O trabalho realizado nesse deslocamento.
Portanto, a potência média é a relação entre o trabalho realizado e o

τ
intervalo de tempo para se realizar este trabalho. Assim temos: 79. Um móvel de massa 40kg tem velocidade constante de 90km/h. Num
determinado instante entra numa região rugosa onde o coeficiente de atrito
é igual a 0,2. Determine:
Pot = I. O espaço percorrido pelo móvel na região rugosa até parar;
∆t II. O trabalho realizado pela força de atrito.
Efetuando algumas transformações matemáticas e lembrando a 80. Um corpo de massa 10kg é arrastado ao longo de um plano horizontal
definição de velocidade podemos escrever a potência como: rugoso cujo coeficiente de atrito vale 0,2 por uma força horizontal de
r r intensidade F = 60N durante 20s. Sabendo que o corpo estava inicialmente
Pot = F • v ou Pot = F ⋅ v ⋅ cosθ em repouso e g = 10m/s2, calcule, nesses 20s.
I. O trabalho realizado pela força F e o trabalho da força de atrito;

AUTORIA – PROF. MARCELO CORREIA 10 E-mail: marcelo.correia.fisica@bol.com.br


FÍSICA – MECÂNICA
II. O trabalho realizado pela força peso e pela normal; ENERGIA
III. O trabalho realizado pela força resultante. Uma das grandezas mais importantes da Física é a energia. A energia
81. Determine o trabalho realizado por uma força de 200N num deslocamento é uma grandeza escalar que pode aparecer de formas variadas na natureza tais
de 8m nos seguintes casos: como: térmica, mecânica, luminosa, nuclear, elétrica, etc. podendo alternar sua
forma em transformações mútuas.
I. Força e deslocamento formam um ângulo de 30º;
O nosso propósito, agora, é estudar a energia mecânica – aquela
II. Força e deslocamento formam um ângulo de 90º; associada aos fenômenos reativos a movimento.
III. Força e deslocamento formam um ângulo de 120º. A energia mecânica pode aparecer em duas formas natureza:
82. Um garoto abandona uma pedra de 0,4kg do alto de uma torre de 25m de • Energia Cinética  Associada ao movimento propriamente dito;
altura. Dado g = 10m/s2, calcule o trabalho realizado pela força peso até a • Energia potencial  Associada a tendência do corpo vir a adquirir
pedra atingir o solo. movimento (energia cinética).
83. Uma partícula é lançada obliquamente no vácuo descrendo uma trajetória
parabólica. Sabendo que a altura máxima atingida pela partícula foi de 5m e Energia Cinética EC
que no local a aceleração da gravidade vale g = 10m/s2. Nestas condições A energia cinética é o tipo de energia mecânica assoada ao
encontre o trabalho, em joule, realizado pela força peso desde o instante de movimento, isto é, um corpo tem energia cinética em relação a um determinado
lançamento até o instante em que partícula passa pelo mesmo nível de referencial se estiver em movimento em relação a este referencial. A energia
lançamento. cinética é dada por:
(a) Zero (b) 5,0 (c) 50 (d) 100 (e) 200 1
r ) )
84. Um bloco sofre um deslocamento d = 15 ⋅ i − 10 ⋅ j em unidade do SI sobre a EC = ⋅ m ⋅ v2
r ) )
ação de uma força F = 220 ⋅ i − 100 ⋅ j em unidade do SI. Calcule o trabalho 2
realizado por esta força no deslocamento considerado. Pela própria definição da energia cinética constatamos um teorema
muito importante chamado de: teorema do trabalho–energia cinética ou
85. Uma usina hidrelétrica foi construída para aproveitar uma queda-d’água de simplesmente teorema da energia cinética. O teorema da energia cinética pode
20m de altura. Se a vazão da água é de 1,5— 102m3/s, qual a potência ser enunciado da seguinte forma:
disponível, supondo que não haja perdas. Dados: aceleração da gravidade O trabalho realizado pela força resultante que atua sobre uma partícula é
µ = 1,0— 103kg/m3 e aceleração da gravidade g = 10m/s2. igual à variação da energia cinética da partícula.

τ = ∆E τ = E
86. (Unicamp–SP) Uma hidrelétrica gera 5,0— 109W de potência elétrica Segundo o teorema da energia cinética podemos escrever:
utilizando-se uma queda-d’água de 100m. Suponha que o gerador aproveita
100% da energia da queda-d’água e que a represa coleta 20% de toda a C ou Cf − E Ci
chuva que cai em uma região de 400000km2. Considere que 1 ano tem
32—106 segundos, g = 10m/s2 e µágua = 1,0— 103kg/m3. Energia Potencial EP
I. Qual a vazão de água (m3/s) necessária para fornecer a potência A energia potencial é um tipo de energia armazenada no corpo, que
indicada? por esse motivo pode vir a ser transformada em energia cinética e assim sendo
II. Quantos mm de chuva devem cair por ano nessa região para em movimento. As energias potenciais mecânicas aparecem quando uma força
manter a hidrelétrica operando na potência indicada? conservativa (força peso ou força elástica) realiza trabalho. Assim podemos
87. O nosso organismo converte energia química interna, em trabalho e em considerar:
calor, com uma potência de 100 W, que é denominada a nossa taxa ou • Energia potencial gravitacional – Proveniente da força gravitacional
potência metabólica. (força peso). Esta é dada por:
I.
II.
Que quantidade de energia dissipamos em 24h?
A energia que dissipamos é oriunda de alimentos, sendo 1kcal =
E Pg = m ⋅ g ⋅ h
4,18kJ. Quantas quilocalorias de alimentos devemos ingerir em 1
semana em devido a taxa metabólica? • Energia potencial elástica – Proveniente da força elástica (lei de
Hooke).
BRINCANDO COM A COVEST (UFPE – UFRPE) 1
88. Um elevador é puxado para cima por cabos de aço com velocidade E Pel = ⋅ k ⋅ x 2
constante de 0,5m/s. A potência mecânica transmitida pelos cabos é de 2
23kW. Qual a força exercida pelos cabos? Análogo ao teorema da energia cinética podemos considerar um
(a) 5,7— 104N (b) 4,6— 104N (c) 3,2— 104N teorema para a energia potencial no caso de só atuar forças conservativas na
(d) 1,5— 104N (e) 1,2— 104N partícula, este pode ser enunciado da seguinte forma:
89. Uma usina hidrelétrica de 90MW produz energia elétrica por meio de uma O trabalho realizado pelas forças conservativas sobre uma partícula é igual
turbina acionada pela água que cai de uma cachoeira cuja altura é 100m. ao negativo da variação da energia potencial desta partícula.

τ = − ∆E
Supondo que não há perdas, calcule o volume de água, em m3, que passa Assim temos:
pela turbina em cada segundo.
P
90. Um bloco de massa m = 1,0g é arremessado horizontalmente ao longo de
uma mesa, escorrega sobre a mesma e cai livremente, como indica a figura.
A mesa tem comprimento d = 2,0 m e altura h = 1,0 m. Qual o trabalho Energia Mecânica EM
realizado pelo peso do bloco, desde o instante em que foi arremessada até Diante do que foi discutido, podemos falar sobre a energia mecânica.
o instante em que toca o chão? Como vimos temos duas formas possíveis de energia mecânica: energia
cinética e energia potencial gravitacional ou elástica, pois bem. A energia
(a) 1,0— 10–2J
mecânica é a soma da energia cinética com a energia potencial. Assim temos:
(b) 1,5— 10–2J
(c) 2,5— 10–2J d EM = E C + EP
(d) 4,0— 10 J
–2 d 2,0m
(e) 5,0— 10–2J
AUTORIA – PROF. MARCELO CORREIA 11 E-mail: marcelo.correia.fisica@bol.com.br
FÍSICA – MECÂNICA
Princípio da Conservação da Energia Mecânica 95. (FUVEST–SP) Um corpo está preso nas extremidades de duas molas
Num sistema em que só atua forças conservativas a energia mecânica idênticas, não deformadas, de constante elástica 100 N/m, conforme ilustra
é conservada, isto é, a energia mecânica é constante. Neste caso podemos a figura. Quando o corpo é afastado de 1,0cm do ponto central:
escrever que:

∆E M = 0 ou EMf − E Mi = 0 ou EMi = EMf


Teorema do Trabalho–Energia
O teorema do trabalho–energia relaciona o trabalho realizado por I. Qual a intensidade da resultante das forças que as molas exercem
forças não–conservativas com a variação de energia cinética e potencial do sobre ele?
sistema. II. Qual a energia potencial armazenada nas molas?
Se num sistema só atua forças conservativas, quando há desaparição
96. (ITA–SP) A variação da energia cinética de uma partícula em movimento,
de energia cinética esta aparece em forma de energia potencial gravitacional ou
num dado referencial inercial, entre dois pontos distintos P e Q é sempre
elástica e quando há desaparição de energia potencial gravitacional ou elástica
igual:
esta aparece na forma de energia cinética, assim, para um sistema em que só
atua forças conservativas a soma da variação da energia cinética com a variação I. À variação da energia potencial entre esses dois pontos.
da energia potencial deverá dá zero. II. Ao trabalho da resultante das forças aplicadas à partícula para
No entanto, se num sistema em que atua forças não–conservativas deslocá-la entre esses dois pontos.
(forças dissipativas) a soma da variação da energia cinética com a energia III. À variação da energia potencial entre esses dois pontos, a menos
potencial deverá da um valor diferente de zero. O Teorema do trabalho–energia de sinal, quando a força resultante aplicada à partícula for
diz que: conservativa.
O trabalho realizado por forças não–conservativas sobre uma partícula é (a) Somente I é correta. (b) I e II são corretas.
igual a soma da variação da energia cinética com a energia potencial da (c) Somente III é correta. (d) II e III são corretas.
partícula, isto é, é igual a variação da energia mecânica da partícula. (e) Somente II é correta.
Observe que a soma da energia cinética com a energia potencial da 97. (ITA–SP) Uma partícula P move-se em linha reta em torno do ponto x0. A
partícula é na verdade a variação da energia mecânica da partícula. Assim, figura ilustra a energia potencial da part´cula em função da coordenada x do

τ τ
podemos escrever: ponto. Supondo que a energia total
Energia
NC = ∆E M ou NC = ∆E + ∆ E P
C
da partícula seja constante e igual
a E, podemos afirmar que:
EP
Onde o índice NC na indica não–conservativa. (a) Nos pontos x1 e x2 a energia E
cinética da partícula é
BRINQUEDINHO DE VESTIBULANDO!!!!! máxima.
91. (ITA–SP) Uma partícula é deslocada de um ponto A até outro ponto B, sob a (b) A energia cinética da
ação de várias forças. O trabalho realizado pela força resultante, nesse partícula entre x1 e x2 é
deslocamento, é igual à variação da energia cinética da partícula: constante.
(a) Somente se a força for constante. (c) No ponto x0 a energia cinética
(b) Somente se a força for conservativa. da partícula é nula.
(d) Nos pontos x1 e x2 a energia 0 x1 x0 x2 x
(c) Seja a força conservativa ou não.
(d) Somente se a trajetória for retilínea. cinética da partícula é nula.
(e) Em nenhum caso. (e) Nenhuma das anteriores.
92. (FEI–SP) Um corpo de massa m = 30 kg, inicialmente em repouso, é posto
(F.M. Santa Casa–SP) Este enunciado refere-se às três próximas
em movimento sob a ação de uma força constante e adquire, ao fim de dois
questões.
minutos, uma velocidade de 72 km/h na direção do da força aplicada.
O gráfico representa a energia potencial de um sistema conservativo
Determine:
isolado, em função da distância x. Para x = 0 o sistema possui só energia
I. A intensidade da força aplicada ao corpo; potencial.
II. O trabalho realizado pela referida força ao longo da distância EP (J)
percorrida pelo corpo. 2M
102
93. (FEI–SP) Um corpo de massa 1,0kg está parado num plano inclinado.
Abandonado, desce e tem a velocidade de módulo 2,0m/s, quando a altura
de queda (vertical) é 0,80m. Calcular o trabalho das forças não– 1— 102
conservativas. Dado: g = 10m/s2.
94. (Mackenzie–SP) Uma bomba (B) recalca
água à taxa de 0,02m3/s, de um 0 1 2 3 4 5 6 7 8 |x| (cm)
depósito (A) para uma caixa (C) no topo
de uma casa. A altura de recalque é C
9,2m e a velocidade da água na D –1— 102
extremidade do topo de descarga (D) É
4m/s. Considerar g = 10m/s2 e a massa 9,2m 98. Para x = 2cm.
específica da água 1000kg/m3. (a) O sistema tem 1,0— 102 joules de energia total.
A
Desprezar as dissipações de energia. A (b) O sistema só tem energia cinética.
potência da bomba é: (c) O sistema tem energia cinética igual à energia potencial.
(a) 2500W (b) 2000W (c) 1500W (d) O sistema perdeu energia.
B
(d) 1000W (e) 500W (e) Nada do que se afirmou é correto.
AUTORIA – PROF. MARCELO CORREIA 12 E-mail: marcelo.correia.fisica@bol.com.br
FÍSICA – MECÂNICA
99. Marque a correta: 106. Com que velocidade a esfera deve passar pelo ponto A para chegar ao
(a) Entre 5cm e 7cm o sistema executa um movimento circular. ponto B com velocidade de 2 5 m/s? Sabe-se que no percurso AB houve
(b) Para |x| maior que 8cm, a energia cinética do sistema é igual a 0,5— 102 J. perda de energia de 20% e g = 10m/s2.
(c) Para x = 6cm o sistema tem certamente energia cinética menor que 2— 102 J.
(d) O sistema perdeu energia.
(e) Nada do que se afirmou é correto nas afirmativas anteriores.
100. Para x = 2cm e x = 6cm: B 3m
(a) As energias potenciais são iguais em valor absoluto. A
(b) As energias cinéticas são iguais.
(c) A soma das energias cinética e potencial variou.
(d) As energias cinética são iguais em módulo.
BRINCANDO COM A COVEST (UFPE – UFRPE)
(e) Nada do que se afirmou é correto.
107. Num circo, um motociclista deve percorrer o perímetro interno de um
101. (Fuvest–SP) Uma bola de 0,2kg é chutada para o ar. Sua energia
trilho circular vertical, de raio R = 4 m, partindo do repouso, de um ponto
mecânica, em relação ao solo, vale 50J. Quando está a 5 m do solo, o valor
com altura H. Se a velocidade do motociclista, ao passar pelo ponto A é de
de sua velocidade é: Dado g = 10m/s2.
10 m/s, qual o valor de H, em metros? Despreze o atrito e considere
(a) 5m/s (b) 10m/s (c) 50 m/s (d) 20m/s (e) 100m/s g=10m/s2.
102. (PUC–SP) Um menino desce um tobogã de altura h = 10m, a partir do A
repouso. Supondo g = 10m/s2 e que sejam dissipados 50% da energia
adquirida na queda, a velocidade do menino ao atingir a base é de: H R
(a) 10 2 m/s (b) 10m/s (c) 5 2 m/s (d) 5m/s (e) 1m/s
103. (Fatec–SP) Na figura, o corpo de 0,2kg é lançado do repouso pela
mola M de constante elástica 6— 103 N/m e descreve a trajetória D, E, F, G,
H e I sem perder o contanto com a trajetória. Adote g = 10m/s2 e despreze 108. Um carrinho escorrega sem atrito em uma montanha russa, partindo do
os atritos. A mínima compressão da mola para que isso ocorra é de: repouso no ponto A, a uma altura H, e sobe o trecho seguinte em forma de
(a) 0,5 cm um semicírculo de raio R. Qual a razão H/R, para que o carrinho permaneça
(b) 1,0 cm M G 10cm em contato com o trilho no ponto B?
H (a) 5/4 A
(c) 5 cm F (b) 4/3
(d) 1,0 m 10cm D B
(c) 7/5
(e) 5,0 m E (d) 3/2 H
R
(e) 8/5

104. Um anel de massa 0,80kg está ligado a uma mola e desliza sem atrito 109. Um garoto desliza sobre um escorregador, sem atrito, de 5,0 m de
ao longo de um guia circular, situada num plano vertical. A constante altura. O garoto é lançado em uma piscina e entra em contato com a água a
elástica da mola é de 40 N/m. uma distância horizontal de 2,0 m, em relação à borda. Calcule a distância
Abandona-se o anel em vertical h, entre a superfície da água e a borda da piscina. Dê sua resposta
repouso na posição A, em cm.
determine sua velocidade
ao passar pelo ponto B. 0,30m A
Sabe-se que o
comprimento da mola,
5,0 m

quando não deformada, é 0,40m


de 0,40m. Considere a B
aceleração da gravidade
10m/s2.
(a) 2m/s (b) 20 m/s (c) 2,8 m/s (d) 0,25 m/s (e) 2 km/h h
105. Um pêndulo de comprimento L=10dm tem um peso de massa m. O 2,0 m
pêndulo é solto fazendo um ângulo θ=60º com a vertical. O fio do pêndulo
bate num pino, colocado 110. Um bloco cai, a partir do repouso, de uma altura h = 0,9 m acima da
à uma distância Z=5dm, extremidade livre de uma mola de
vertical, do ponto onde constante elástica k= 4,2x103 N/m,
está vinculado o como mostra a figura. Se a
pêndulo no teto o que θ
L Z deformação máxima da mola é
provoca uma diminuição h
x = 0,1 m, qual o peso do bloco,
do comprimento do
em newtons. Despreze a
pêndulo. Achar o ângulo
resistência do ar e a massa da
máximo β entre o fio e a
mola. x
vertical, quando o peso
do pêndulo estiver à β K
direita do pino,
conforme a figura.

AUTORIA – PROF. MARCELO CORREIA 13 E-mail: marcelo.correia.fisica@bol.com.br


FÍSICA – MECÂNICA
111. Deixa-se cair uma bola, a partir do repouso, de uma altura H acima do 116. Saltando de um helicóptero, estacionário, um pára-quedista ganha 30J
piso de uma quadra. Após a bola colidir três vezes com o piso, ela se eleva de energia cinética após um determinado tempo de queda. Considerando a
até uma altura H' = H/8. Considerando que a razão entre as energias perda de energia no movimento através do ar, a variação da energia
cinéticas antes e depois de cada colisão, é a mesma, determine o valor potencial gravitacional, neste mesmo intervalo de tempo, é:
desta razão? Despreze a resistência do ar. (a) Igual a variação da energia cinética.
112. Um bloco de massa m = 0,1 kg comprime uma mola ideal, de (b) Menor que 30 J.
constante elástica k = 100 N/m, de 0,2 m (ver figura). Quando a mola é (c) Maior que 30 J.
liberada, o bloco é lançado ao longo de uma pista lisa. Calcule a velocidade (d) Igual ao trabalho das forças não conservativas.
do bloco, em m/s, quando ele atinge a altura h = 1,2 m. (e) Sempre igual ao dobro do trabalho das forças não conservativas.
117. Uma menina de massa M é empurrada no topo de um escorregador,
k m partindo com uma velocidade inicial vo de ma altura H, conforme figura ao
h = 1,2 m lado. Desprezando a ação das forças não conservativas, é correto afirmar
que a velocidade da menina, ao atingir a altura h, é:
0,2 m 2
(a) V0 + 2g(H − h)
113. A figura mostra um bloco de 0,10kg inicialmente forçado contra uma
(b) 2gH
mola de constante elástica k = 480N/m, comprimindo-a de 10cm. Ao soltar,
o bloco desliza sobre uma superfície horizontal lisa, exceto no trecho AB, de 2
50cm, onde o coeficiente de atrito cinético é igual a 0,25. Em seguida o (c) V0 + 2gH
bloco sobe uma rampa sem atrito, retornando posteriormente à superfície
horizontal podendo atingir a mola. Quantas vezes o bloco passará pelo (d) V0 + 2gH
ponto A antes de parar completamente?
(a) 20 (e) V0 + 2g(H − h)
(b) 9 k
(c) 18 118. Em relação ao conceito e ao tipo de força.
(d) 24 I II
A B
(e) 25 0 0 As forças de ação e reação sempre atuam em corpos distintos.
1 1 A força elástica é proporcional à deformação da mola.
114. Um brinquedo consiste de duas peças de plástico ligadas através de
2 2 A força normal é uma força de reação ao peso.
uma mola. Quando pressionado sobre o solo e abandonado, ele sobre
verticalmente na direção da normal. O centro de massa do brinquedo atinge 3 3 Força é uma grandeza vetorial.
uma altura máxima de 50,0 cm, quando a compressão inicial da mola é de 4 4 Uma força sempre causa mudança no valor da velocidade.
2,0cm. Se a massa total do brinquedo vale 200g, quanto vale a constante 119. Um carrinho de montanha-russa, de massa igual a 20kg, parte do
elástica da mola? repouso no ponto A, a uma altura HÁ = 50m e, após percorrer o trilho
(a) 1,0— 103N/m indicado na figura, alcança a altura máxima HC = 45m, no ponto C, de onde
(b) 2,0— 103N/m retorna. Analise as proposições apresentadas.
(c) 3,0— 103N/m
(d) 4,0— 103N/m A
C
(e) 5,0— 103N/m
HA HC
B

I II
BRINCANDO COM A UPE A variação da energia potencial do carrinho entre os pontos A
0 0
115. Um carro de 700 kg tem velocidade de 20 m/s quando está em x = 0. e C é igual a 1000 joules.
Sobre o carro atua F(N) O carrinho tem a mesma velocidade, ao passar pelo ponto B,
1 1
uma única força F(x) na subida e na descida.
7000 Se o percurso total AC tem 125m de comprimento, a força
que varia com a 2 2
posição, como média de atrito que atuou sobre o carrinho foi igual a 8 N.
mostra o gráfico. A soma das energias cinética e potencial gravitacional é
3 3
Em relação a esta sempre decrescente ao longo do caminho AC.
situação, analise as 4 4 Ao retornar, o carrinho não conseguirá atingir o ponto A.
0 50 x(m)
propostas abaixo:
I II
0 0 A energia cinética do carro em x = 0, é 140 kJ.
O trabalho realizado pela força, quando o carro se desloca de
1 1
x = 0 até x = 50 m, é 175kJ.
2 2 A velocidade do carro em x = 50 m é 30 m/s.
3 3 A energia mecânica é sempre menor do que 140kJ.
4 4 A variação da energia potencial é diferente de zero.

AUTORIA – PROF. MARCELO CORREIA 14 E-mail: marcelo.correia.fisica@bol.com.br


FÍSICA – MECÂNICA
DINÂMICA – PARTE 3 Cálculo do Momento Linear de um Sistema de Partículas
Nesta terceira parte do estudo da dinâmica vamos estudar grandezas Um sistema de partículas nada mais é do que um conjunto de
físicas muito importantes: quantidade de movimento (que também pode ser partículas.
chamada de momento linear) e impulso. Vamos discutir um princípio de Considere um conjunto de “n” partículas. Cada partícula tem a sua
conservação muito importante: O princípio de conservação do momento linear. massa e sua velocidade vetorial, assim:
r
Nesta parte vamos fazer uma introdução para o estudo de um sistema de • A partícula 1 tem massa m1 e velocidade vetorial v 1 ;
partículas, na verdade, iremos ver como calcular o momento linear de um sistema r
• A partícula 2 tem massa m2 e velocidade vetorial v 2 ;
de partículas. r
• A partícula 3 tem massa m3 e velocidade vetorial v 3 ;
MOMENTO LINEAR OU QUANTIDADE DE MOVIMENTO r
• A partícula n tem massa mn e velocidade vetorial v n ;
Definimos a quantidade de movimento (ou momento linear) de uma
Assim, cada partícula terá o seu momento linear:
partícula como sendo o produto da massa “m” da partícula pela sua velocidade
r r r
vetorial “ v ”. Assim temos: • A partícula 1 tem momento linear Q1 = m1 ⋅ v1;
Onde: r r
r r r
Q • A partícula 2 tem momento linear Q2 = m2 ⋅ v2 ;

Q = m⋅ v
 Momento linear ou quantidade de r r
movimento • A partícula 3 tem momento linear Q3 = m3 ⋅ v3 ;
m  massa da partícula r r
r
v  Velocidade vetorial • A partícula n tem momento linear Qn = mn ⋅ vn .
Nota: r
• Observe que o momento linear e a velocidade têm a mesma direção e O momento linear do sistema de partículas “ Q ” é dado pela soma dos
sentido; momento lineares das n partículas que constituem o sistema. Assim temos:
• A unidade, no SI, do momento linear é: kg— m/s.
r n r r r r r
IMPULSO
r Q = ∑Qi ou Q = Q + Q +L+ Q
Definimos o impulso como sendo o produto da força “ F ” que atua 1 2 n
sobre a partícula pelo intervalo de tempo “∆t” que esta força atua. Assim temos: i =1
Onde:
r r r
Ir  Impulso
Conservação do Momento Linear (Quantidade de Movimento)

I = F⋅ ∆t
Podemos enunciar o princípio de conservação do momento linear da
F  força seguinte forma:
r
∆t  intervalo de tempo que a força F NUM SISTEMA DE PARTÍCULAS EM QUE NÃO ATUA AGENTE
atua
EXTERNO O MOMENTO LINEAR É CONSERVADO, ISTO É, É
Nota:
• Observe que o impulso e a força têm a mesma direção e sentido; CONSTANTE.
• A unidade, no SI, do momento linear é: N— s.
Assim podemos escrever que:
• Podemos constatar facilmente que a unidade de impulso no SI é
SE NÃO ATUA AGENTE EXTERNO
equivalente a unidade de momento linear no SI, isto é: N— s = kg— m/s. r r
• A observação anterior mostra que existe uma ligação entre as
grandezas vetoriais momento linear e impulso, esta ligação é
Q inicial = Q final
mostrada e expressa em forma de um teorema chamado TEOREMA
DO IMPULSO. Colisões ou Choques
As colisões ou choques são estudados tomando a conservação do
Teorema do Impulso momento linear, por isso é pertinente que fazer referência ao assunto.
O teorema do impulso pode ser enunciado da seguinte forma: Podemos considerar três tipos de colisões, que são:
• Colisão Perfeitamente Elástica  Quando após a colisão os corpos
O IMPULSO DA FORÇA RESULTANTE QUE ATUA SOBRE UMA se separam e não ficam com deformações. Nesta colisão há
PARTÍCULA É IGUAL À VARIAÇÃO DA SUA QUANTIDADE DE conservação do momento linear e da energia cinética do sistema de
MOVIMENTO (MOMENTO LINEAR). partículas que colide entre si;
• Colisão Parcialmente Elástica  Quando após a colisão os corpos
se separam e pelo menos um dos corpos permanece deformado.
Assim temos:
r r Nesta colisão há conservação do momento linear e há dissipação de

I = ∆Q
energia cinética, isto é, a energia cinética depois da colisão é menor
do que a energia cinética antes da colisão;
• Colisão Perfeitamente Inelástica  Quando após a colisão os
corpos se deslocam juntos. Nesta colisão há conservação do
Gráfico da Força em Função do Tempo momento linear e a há uma dissipação máxima de energia cinética em
No gráfico da força em função do tempo o impulso é dado relação aos outros dois tipos de colisões.
(numericamente) pela área limitada pelo gráfico e o eixo dos tempos. Nota:
F n Observe que em todos os tipos de colisão há conservação do
A t
 I=A momento linear.

x
0
AUTORIA – PROF. MARCELO CORREIA 15 E-mail: marcelo.correia.fisica@bol.com.br
FÍSICA – MECÂNICA
BRINQUEDINHO DE VESTIBULANDO!!!!! 130. O gráfico abaixo representa a variação da velocidade com o tempo de
120. Um carrinho de supermercado, carregando latas de óleo, foi um objeto de massa igual a 10kg que se desloca em linha reta. Qual a
abandonado numa rampa e adquiriu uma velocidade escalar constante de variação do momento linear do objeto, em kg— m/s, a cada intervalo de 2s?
50cm/s. Sabendo que sua massa total é de 20kg, qual o módulo da (a) 20 F(N)
quantidade de movimento do carrinho, em kg— ms? (b) 40 7
(a) 1 (b) 12 (c) 15 (d) 10 (e) 20 (c) 60 6
121. Duas partículas 1 e 2 têm massas iguais, e suas respectivas energias (d) 80 5
cinéticas Ec1 e Ec2 são tais que Ec1 = 4Ec2. Calcule a razão entre os módulos (e) 100 4
de suas quantidades de movimento. 3
2
122. Uma partícula de massa m = 10kg realiza um MCU com velocidade de 1 t(s)
módulo 1,0m/s. Determine o módulo da variação de seu momento linear nos
0 1,0 2,0
seguintes intervalos de tempo:
I. Um quarto de período; 131. Um jogador de tênis pode sacar a bola com velocidade de 50m/s.
II. Meio período; Sabendo que a massa de uma bola de tênis é 60g, calcule o impulso (em
III. Três quartos de período; unidades do sistema MKS) fornecido a bola quando ela é sacada.
IV. Um período. 132. A figura mostra a
123. (FCMSC–SP) Em uma carta de Benjamin Franklin, como objeto à variação no tempo da
intensidade de uma força F F(N)
teoria corpuscular da luz, ele declarava:
“Uma partícula de luz, caminhando com a velocidade da luz, deveria que atua sobre um corpo de 20
produzir o mesmo impacto que uma bala de canhão e massa 10kg animada massa igual a 2,5kg.
de velocidade de 300m/s, ao atingir a superfície da Terra”. Sabendo que em t = 0 o t(s)
Nestas condições, a partícula de luz a que se referia Franklin deveria ter corpo estava em repouso, e
massa, expressa em kg, de ordem de grandeza igual a: que a força tem direção 0 2,0 4,0 6,0 8,0 10,0
(a) 10–8 (b) 10–6 (c) 10–5 (d) 10–7 (e) 10–4 constante, calcule a –10
velocidade do corpo no
124. Num jogo de vôlei, um jogador, ao efetuar uma cortada, aplicada na instante t = 10s.
bola um impulso de intensidade de 80N— s. Se a duração da cortada é de
0,20s, determine a intensidade média da força que o jogador aplica na bola.
133. Um átomo de argônio, movendo-se com velocidade igual a 400m/s,
choca-se elasticamente contra a parede de um recipiente. Se a massa do
átomo de argônio é de 6,5— 10–26kg, qual o impulso sobre a parede, durante
BRINCANDO COM A COVEST (UFPE – UFRPE)
o impacto, em unidades de 10–24N?
125. Uma criança de massa igual a 30kg e um homem de massa igual a
60kg estão parados em pé, um em frente ao outro, numa pista de patins. De 134. Um patinador de 65 kg, em repouso, arremessa um peso de 5,0 kg,
repente, o homem empurra a criança para trás com uma velocidade de horizontalmente para frente. A velocidade do peso em relação ao patinador
1m/s. Pode-se afirmar que a velocidade de recuo do homem será, em m/s, é de 3,5 m/s no instante do arremesso. Calcule o módulo da velocidade em
igual a: relação à Terra, adquirida pelo patinador, em cm/s. Despreze o atrito entre
(a) 0,5 (b) 0,8 (c) 1,0 (d) 1,5 (e) 2,0 os patins e o piso.
126. Um menino, sentado numa canoa parada na superfície de um lago, v V
E
atira um tijolo par fora. A massa do menino e da canoa é, no total, 40kg.
Sabendo que a massa do tijolo é 0,4kg, e que sua velocidade, ao sair da
mão do menino, é de 10m/s em relação à água, qual é a velocidade, em
cm/s, com que a canoa começa a se movimentar?
(a) 12 (b) 15 (c) 10 (d) 8 (e) 20 135. Um rapaz de 59 kg está parado sobre um par de patins, no instante em
127. Um vagão corre ao longo de trilhos horizontais, sem atrito. Em que ele pega um pacote de 1,0 kg que foi jogado em sua direção. Depois de
determinado instante, colide com outro, de igual massa, que estava apanhar o pacote, o rapaz recua com uma velocidade igual a 0,3 m/s. Qual
inicialmente parado. Os dois vagões passam então a se mover unidos. Se a a velocidade horizontal do pacote, em m/s, imediatamente antes de ele ser
energia cinética inicial do primeiro vagão era igual a 40kJ, quanto deve apanhado? Despreze o pequeno atrito do solo com as rodas dos patins.
valer, em kJ, a energia cinética dos dois vagões após a colisão?
136. Um bloco de massa m1 = 100 g comprime uma mola de constante
128. (Modificada) Durante o jogo de futebol, professores versus alunos do elástica k = 360 N/m, por uma distância x = 10,0 cm, como mostra a figura.
MEGA, o professor Eriberto é atingido frontalmente por uma bola de massa Em um dado instante, esse bloco é liberado, vindo a colidir em seguida com
500g, a uma velocidade de 72km/h. Qual a força média, em 103N, que atua um outro bloco de massa m2 = 200 g, inicialmente em repouso. Despreze o
no rosto do professor Eriberto durante o impacto, se a bola retorna no atrito entre os blocos e o piso. Considerando a colisão perfeitamente
sentido oposto com a mesma velocidade em módulo, e a colisão teve a inelástica, determine a velocidade final dos blocos, em m/s.
duração de 0,01s?
129. Uma força aplicada durante 1s a um objeto de massa 10kg varia de m1 m2
intensidade conforme o gráfico abaixo. Qual o impulso total da força, em k
N—s, após a interação? F(N)
(a) 10 20
(b) 15
(c) 20
10 10 cm
(d) 25 t(s)
(e) 30
0 0,5 1,0

AUTORIA – PROF. MARCELO CORREIA 16 E-mail: marcelo.correia.fisica@bol.com.br


FÍSICA – MECÂNICA
DINÂMICA – PARTE 4 BRINCANDO COM A COVEST (UFPE – UFRPE)
Nesta quarta parte do estudo da dinâmica vamos estudar sistemas de 137. A figura representa a molécula de NO. O átomo de nitrogênio tem
partículas e corpo rígido: centro de massa de um sistema de partículas será o massa atômica 14 e o átomo de oxigênio tem massa atômica 16. A distância
nosso primeiro objeto de estudo seguido das grandezas dinâmicas que estão entre os dois átomos é D = 1,5— 10–8cm. Qual a distância entre o centro de
envolvidas no estudo da rotação de um corpo rígido (energia cinética massa da molécula e o átomo de nitrogênio, em unidades de 10–10cm?
rotacional, momento de inércia, momento angular e conservação do momento N O
angular). Esta parte da dinâmica é muito importante para a 2ª fase da Covest e
principalmente para UPE. D

CENTRO DE MASSA
Um sistema mecânico pode ser formado por um número muito grande 138. A figura mostra uma estrutura vertical formada por três barras iguais,
de partículas, é o que chamamos de sistema de partículas. O que chamamos de homogêneas e de espessuras desprezíveis. Se o comprimento de cada
corpo rígido é nada mais do que um conjunto infinito de partículas, tal como uma barra é 90cm, determine a altura, em cm, do centro de massa do sistema,
cadeira. em relação ao solo.
O movimento das diversas partículas que constituem o nosso sistema,
em diversos casos, poderá ser estudado considerando-se o movimento de um
único ponto em que consideramos que a massa de todas as partículas está 90cm
concentrada neste ponto. A este único ponto chamamos de centro de massa.
Para localizar o centro de massa devemos adotar um sistema de
referência (sistema cartesiano) e aplicar as equações mostradas a seguir:
n

∑m ⋅ x i i
139. Uma chapa de aço, uniforme, é cortada na forma indicada na figura
abaixo. À que distância do ponto P, em cm, está localizado o centro de
xCM = i=1 n m1 ⋅ x1 +m2 ⋅ x2 +L+ mn ⋅ xn
ou xCM = massa da chapa?
m1 + m2 +L+ mn
∑m
i=1
i
9 2

18 2
n
9 2
∑m ⋅ y i i
yCM = i=1
n ou yCM =
m1 ⋅ y1 + m2 ⋅ y2 +L+ mn ⋅ yn 18 2

∑m
i=1
i
m1 + m2 +L+ mn
140. Duas partículas, de massas M1 = M e M2 = M/2, estão presas por uma
haste de comprimento L = 48cm e massa desprezível, conforme a figura.
Qual à distância, em centímetros, do centro de massa do sistema em
n relação à posição da partícula de massa M1?
∑m ⋅ z i i
M2
zCM = i=1 m1 ⋅ z1 + m2 ⋅ z2 +L+ mn ⋅ zn
n ou zCM = M1
L
m1 + m2 +L+ mn
∑m i=1
i 141. Uma caixa cúbica, sem tampa, com 40cm de lado, é feita de chapas
de metal homogêneas de espessura desprezível. Determine a localização
do centro de massa da caixa em relação ao fundo (com tampa) e duas de
A figura abaixo ilustra a aplicabilidade do que foi exposto suas faces laterais.
anteriormente: 142. A figura mostra as dimensões de uma placa composta; metade da
z placa é feita de alumínio (densidade = 2,70g/cm3) e metade ferro
(densidade = 7,85g/cm3). Qual a localização do centro de massa da placa
z1
mn em relação ao seu centro geométrico?
m1 24,0cm
z2
z3
zn yn
m3 12,0cm
m2
x3 x1 x2
Ferro 2,0cm
xn x Alumínio
y3
y1
y2
12,0cm
y 12,0cm

AUTORIA – PROF. MARCELO CORREIA 17 E-mail: marcelo.correia.fisica@bol.com.br


FÍSICA – MECÂNICA
ROTAÇÃO DO CORPO RÍGIDO 145. Considerando o sistema de partículas da figura encontre:
É importante lembrar das grandezas cinemáticas que descrevem o I. O momento de inércia do sistema de partículas em relação ao
movimento circular, portanto você pode fazer uma revisão nas páginas 26 e 27 eixo dos y que passa pelas partículas m3 e m4;
na parte de cinemática para recordar estas grandezas. Um corpo rígido em II. O momento de inércia do sistema de partículas em relação ao
rotação pura é um conjunto de partículas descrevendo movimento circular e eixo z que passa por m1 e m4;
sendo assim as grandezas angulares são mais apropriadas para o seu estudo.
III. O momento de inércia do sistema de partículas em relação ao
Sabendo que já foram discutidas as grandezas cinemáticas que
eixo dos x que passa por m4 e é perpendicular ao plano do
precisamos para entender a rotação, que são basicamente: posição angular,
quadrado.
velocidade angular, aceleração angular e aceleração vetorial (tangencial e
centrípeta), vamos nos preocupar com as grandezas dinâmicas. z

ENERGIA CINÉTICA ROTACIONAL m1 m2


Um corpo em rotação tem energia cinética, cada partícula que constitui
o corpo tem energia cinética que é dada pela nossa conhecida expressão: m1 = 3 kg
m2 = 4 kg
EC = ½ m— v2. Assim, a energia cinética do corpo é a soma das energias cinéticas
m3 = 3 kg
de todas as partículas que constituem o corpo, logo, temos:
2,0m m4 = 4 kg
2 2 2
m ⋅v m ⋅v m ⋅v
ECRot = 1 1 + 2 2 +L+ n n m4 m3 y
2 2 2
O problema na expressão acima é a de que cada partícula tem uma 2,0m
velocidade escalar diferente dificultando o cálculo da energia cinética. Podemos,
no entanto, resolver este problema se substituirmos a velocidade escalar de cada
x
partícula pela sua corresponde velocidade angular, pois sabemos que todas as
partículas do corpo em rotação pura têm a mesma velocidade angular, isto pode
ser feito lembrando que: v = ω— R, onde: ω  velocidade angular e R  é o raio 146. A figura a seguir mostra um sistema constituído por oito partículas
da trajetória descrita pela partícula (é a distância entre a partícula e o eixo de idênticas, todas de massa m = 4 kg. Considerando que as barras que ligam
rotação do corpo rígido). Fazendo a substituição, temos: as partículas tem massas desprezíveis encontre:
m1 ⋅ (ωr1) m2 ⋅ (ωr2 ) m ⋅ (ωr )
2 2 2
I. Inércia rotacional do sistema de partículas em relação ao eixo dos
ECRot = + +L+ n n x;
2 2 2 II. A inércia rotacional do sistema de partículas em relação ao eixo

E CRot =
1 2
2
( 2 2
ω ⋅ m 1 ⋅ r1 + m 2 ⋅ r2 + L + m n ⋅ rn
2
) III.
dos y;
A inércia rotacional do sistema de partículas em relação ao eixo
dos z; z
( )
n
1
E CRot = ω 2 ⋅ ∑ m i ⋅ ri
2

2 i =1

( ) que é
n
I = ∑ m i ⋅ ri
2
Na expressão acima destacamos o fator:
i =1
2m
chamado de MOMENTO DE INÉRCIA ou ENÉRCIA ROTACIONAL do corpo em
y
relação ao eixo de rotação. Substituindo na equação da energia cinética que
encontramos, teremos finalmente:
8m
1
E CRot = I ⋅ ω2
2
Observe que a inércia rotacional desempenha um papel na rotação
análogo ao papel desempenhado pela massa na translação. Observe que a x 6m
expressão para a energia cinética de um corpo é dada pela soma de sua energia
cinética de translação com a sua energia cinética de rotação, assim:
E CTotal = E CTrans + E CRot 147. Com relação a questão anterior, considere que o sistema de partícula
está em rotação cujo eixo de rotação é o eixo dos x. Sabendo que a
1 1
E CTotal = m ⋅ v 2 + I ⋅ ω 2 velocidade angular do sistema de partículas vale 8 rad/s calcule sua
2 2 energia cinética rotacional.
148. Com relação a questão 146, considere que o sistema de partícula está
BRINQUEDINHO DE VESTIBULANDO!!!!! em rotação cujo eixo de rotação é o eixo dos y. Sabendo que a velocidade
143. Calcule o momento de inércia de uma roda que tem 24400J de energia angular do sistema de partículas vale 8 rad/s calcule sua energia cinética
rotacional.
cinética de rotação ao girar a 600rpm. Considere π = 3,14.
149. Com relação a questão 146, considere que o sistema de partícula está
144. Um disco de 8kg raio igual a 0,5m gira com uma velocidade na em rotação cujo eixo de rotação é o eixo dos z. Sabendo que a velocidade
periferia de 4m/s. Sabendo que o seu momento de inércia é dado por: angular do sistema de partículas vale 8 rad/s calcule sua energia cinética
1 2 rotacional.
I= ⋅ m ⋅ r , encontre a sua energia cinética rotacional.
2
AUTORIA – PROF. MARCELO CORREIA 18 E-mail: marcelo.correia.fisica@bol.com.br
FÍSICA – MECÂNICA
TORQUE OU MOMENTO DE UMA FORÇA Escrevendo a 2ª Lei de Newton para a Rotação
Para provocar rotação em um corpo, a força aplicada não depende Não é difícil perceber que o torque desempenha na rotação o mesmo
apenas da intensidade ser suficiente, mas de onde é aplicada. papel que a força representa na translação, portanto podemos escrever a
Considere o corpo, na figura abaixo, e observe que as forças F1 e F2 segunda lei de Newton em termos de torque para a rotação. Assim podemos
podem provocar rotação em torno do eixo de rotação 0, porém as forças F3 e F4, mostrar que:

Mres =I ⋅α
por mais intensas que sejam, não provocam rotação.
F3
F1
Eixo de θ1 Na expressão acima observamos que o módulo do torque resultante é
rotação r1 F4 o momento de inércia ou inércia rotacional (análogo da massa para a rotação)
multiplicado pela aceleração angular (aceleração adequada para descrever a
0 r4 variação do momento para a rotação).
r2 Notas:
• A unidade de torque no sistema internacional é o N— m que não deve
θ2 ser confundido com a unidade de trabalho N— m = J (joule). Pois estas
duas grandezas e suas unidades são completamente diferentes;
F2 • O torque resultante é a soma vetorial de todos os torques que atuam
no corpo. Cada força que a tua no corpo provoca um torque (incluindo
Na figura anterior os vetores r1, r2, r3 (não aparece porque seu módulo a possibilidade de ser nulo), assim, a soma vetorial destes torques é o
é zero) e r4 são chamados de braço e indicam onde as forças estão sendo torque resultante, tal como a soma de todas as forças que atuam num
aplicadas em relação ao ponto 0 (eixo de rotação). Observe que a força F3 não corpo é a força resultante.
provoca rotação, isto é, não provoca torque porque está aplicada no próprio eixo
de rotação: imagine que você vai abrir uma porta e aplica a força onde está a MOMENTO ANGULAR OU QUANTIDADE DE MOVIMENTO ANGULAR
dobradiça. Já a força F4 não provoca rotação porque está aplicada na mesma O momento angular é o análogo rotacional do momento linear
direção do braço e sendo assim não provoca torque: imagine que você vai abrir (quantidade de movimento). O momento angular de uma partícula é definido
uma porta e aplica a força ao longo da direção da porta. como sendo o produto vetorial da posição da partícula em relação ao centro de
Na figura seguinte nos concentramos numa força e observamos que a
componente da força que atua no corpo que provoca torque é àquela que é r r r
curvatura da trajetória pelo momento linear da partícula, assim temos:
perpendicular ao braço, que representamos por F┴. A componente da força
tangencial, representada por Ftg, ao braço não contribui para a produção de
torque.
L = r ×Q
Como já sabemos do produto vetorial temos que:
F • Módulo: L = r— Q— senθ, onde θ é o ângulo formado entre a posição
em relação ao centro de curvatura da trajetória e o momento linear, r é
F┴ o módulo da posição e Q é o módulo do momento linear;
Eixo de θ
• Direção e sentido são dados pela regra da mão esquerda que é
rotação r Ftg lembrada na figura a seguir;
• Lembre que o momento linear (ou quantidade de movimento) é a
0 r r
grandeza vetorial definida como: Q = m⋅ v ;
• O momento angular de um sistema de partículas é a soma vetorial do
Observando que F┴ = F— senθ e que o torque é proporcional a esta momento angular de todas as partículas que constituem o sistema;
componente da força e ao braço, podemos definir o torque ou momento de uma • A unidade de momento angular no SI é o kg— m2/s = J— s;
força como sendo um vetor dado pelo produto vetorial entre o braço e a força,
Momento Angular de um Corpo Rígido
assim temos:
r r r Não é difícil mostrar que para um corpo rígido o momento angular é

M = r ×F dado por:


Como já sabemos do produto vetorial temos que:
Módulo: M = r— F— senθ, onde θ é o ângulo formado entre o braço e a
L =I ⋅ω
Isto é, o momento angular é o produto do momento de inércia (ou
força, r é o módulo do braço e F é o módulo da força; inércia rotacional) do corpo pela sua velocidade angular.
• Direção e sentido são dados pela regra da mão esquerda que é
lembrada na figura a seguir. Sabemos que a taxa de variação do mento linear em relação ao tempo
r r r r é igual a força resultante que atua no corpo, analogamente temos que a taxa de

M= r ×F variação temporal do momento angular é igual ao torque resultante, assim

F
podemos escrever que:
r
r ∆L
MRes =
r ∆t
r
AUTORIA – PROF. MARCELO CORREIA 19 E-mail: marcelo.correia.fisica@bol.com.br
FÍSICA – MECÂNICA
Conservação do Momento Angular 158. O rotor de um motor elétrico tem inércia rotacional de 2,0— 10–3kg— m2
Podemos enunciar o princípio de conservação do momento angular da em torno do seu eixo central. O motor é instalado para mudar a orientação
seguinte forma: de uma sonda espacial. O eixo do rotor é instalado de forma paralela ao eixo
da sonda, que tem momento de inércia igual a 24kg— m2 em torno do seu
NUM SISTEMA DE PARTÍCULAS EM QUE NÃO ATUA TORQUE eixo. Encontre o número de revoluções que tem que dar o rotor para fazer a
EXTERNO (RESULTANTE) O MOMENTO ANGULAR É sonda girar um ângulo de 30º em torno do seu eixo.
CONSERVADO, ISTO É, É CONSTANTE.
159. (UPE–2004) Um menino está sentado em uma cadeira que está
Assim podemos escrever que: girando em torno de um eixo vertical, com velocidade angular ù0, conforme
SE NÃO ATUA AGENTE EXTERNO figura. O menino tem os braços estendidos e segura um altere em cada
r r
L inicial = L final
mão, de modo que o momento de inércia do sistema (menino, halteres e
assento) é I0 . O menino abraça rapidamente os halteres, de modo que o
momento de inércia final do sistema reduza de 70% do momento de inércia
BRINQUEDINHO DE VESTIBULANDO!!!!! inicial. Desprezando o torque devido ao atrito no eixo da cadeira, durante o
intervalo de tempo no qual o momento de inércia do sistema varia, é correto
150. Um tubo de paredes finas rola pelo chão. Seu momento de inércia em
afirmar que a velocidade angular final do sistema é aproximadamente:
relação a um eixo paralelo ao seu comprimento e que passa pelo seu centro
de massa é dado por: I = m— R2, em que R é seu raio e m é sua massa.
Encontre a razão entre as sus energias cinética translacional e rotacional,
em torno de um eixo paralelo ao seu comprimento e que passa pelo seu
centro de massa.
151. Uma pequena (tamanho desprezível) bola de 4kg está presa a uma
das extremidades de uma haste de 2m de extensão e de massa
desprezível, a outra extremidade está presa a um eixo no teto, formando um
pêndulo. Quando o pêndulo é desviado 30º da vertical, qual o módulo do
torque em relação ao eixo?
152. Uma pequena (tamanho desprezível) bola de 4,0kg está presa a uma (a) ù0 (b) 3,33— ù0 (c) 1,43— ù0
das extremidades de uma haste de 2,0m de extensão e de massa igual a (d) 0,3— ù0 (e) 0,7— ù0
1,0kg, a outra extremidade está presa a um eixo no teto, formando um 160. Suponha que o combustível do Sol se extinga e ele, subitamente, entre
pêndulo. Quando o pêndulo é desviado 30º da vertical, qual o módulo do em colapso formando um tipo de estrela denominada anã branca, com um
torque em relação ao eixo? diâmetro igual ao da Terra. Considerando que não houvesse perda de
153. Um ciclista de 75kg cola todo o peso de seu corpo em cada um dos massa e que o Sol é uma esfera maciça e homogênea, encontre o novo
pedais, para movimentá-los para baixo. Considerando que a roda da período do Sol nestas condições. Dados: Período do Sol: 25 dias; Massa do
corrente, onde o pedal está preso, tem 40cm de diâmetro e que o pedal é Sol: ≈ 2— 1030kg; Diâmetro do Sol: ≈14— 108m; massa da Terra: ≈ 6— 1024kg;
instalado de forma a ficar rente a periferia da roda da corrente, determine o diâmetro da Terra: ≈ 12— 106m; momento de inércia de uma esfera maciça
módulo do torque máximo que o ciclista pode exercer nesse processo. em relação a um eixo de rotação em que seu diâmetro está contido é dado
154. Uma roda tem uma aceleração angular de 20rad/s2, quando o torque por: I = (2— m— R2)/5, onde: m é a massa da esfera e R é o seu raio.
300N— m é aplicado sobre ela. Qual o momento de inércia da roda? 161. O momento de inércia de uma estrela girando (considere com uma
155. A figura abaixo mostra dois blocos de massas iguais suspensos nas esfera maciça e homogênea) que está em colapso cai a um terço do seu
extremidades de uma haste rígida, sem peso, de comprimento L1 = 20cm e valor inicial. Qual o fator de aumento de sua energia cinética rotacional.
L2 = 80cm. A haste é mantida na horizontal e então é solta. Calcule a
aceleração dos dois blocos quando eles começam a se mover. ESTÁTICA
Trataremos a partir de agora da estática. A estática é a parta da
20cm 80cm mecânica que estuda os corpos em equilíbrio estático e na verdade é um caso
particular da dinâmica.
A estática pode ser dividida em três partes:
• Estática do ponto material;
• Estática do corpo rígido ou do corpo extenso;
• Estática dos fluidos ou Fluidostática ou ainda Hidrostática.

Estática do Ponto Material


A estática do ponto material trata do equilíbrio estático das partículas,
156. Uma chaminé alta, de forma cilíndrica, cai se houver uma ruptura na isto é, dos corpos que podemos desprezar suas dimensões na análise do
sua base. Tratando a chaminé como um bastão fino, de altura h, encontre a fenômeno.
componente tangencial da aceleração linear do topo da chaminé quando A estática do ponto material é basicamente aplicar a condição de
esta formar 30º com a vertical. Sabe-se que o momento de inércia de um equilíbrio de translação, já que não tem sentido tratar de rotação de uma
bastão em relação a um eixo de rotação na extremidade é dado por: partícula. Já foi discutida a condição de equilíbrio de translação que:
I = (m— L2)/3, onde: m é a massa do bastão e L é o seu comprimento. SE UMA PARTÍCULA ESTÁ EM EQUILÍBRIO ESTÁTICO A FORÇA
157. Um projétil de massa m = 2kg é atirado do chão obliquamente RESULTANTE QUE ATUA SOBRE A MESMA É NULA.
formando um ângulo de 60º com a horizontal. Sabendo que a velocidade O que devemos fazer é simplesmente sofisticar esta condição no
inicial de lançamento é 10m/s e a aceleração da gravidade local 10m/s2, sentido de aplicá-la de uma forma mais sistemática. Isto é feito mediante a
encontre o momento angular, em unidades do SI, do projétil 2s após o seu adoção de um sistema de referência retangular (sistema cartesianos) o que nos
lançamento. faz poder escrever a condição de equilíbrio de translação da seguinte forma:

AUTORIA – PROF. MARCELO CORREIA 20 E-mail: marcelo.correia.fisica@bol.com.br


FÍSICA – MECÂNICA
SE UMA PARTÍCULA ESTÁ EM EQUILÍBRIO ESTÁTICO: BRINQUEDINHO DE VESTIBULANDO!!!!!
r
r  ∑ Fx = 0
162. (COVEST) Uma pessoa usa uma corda para atravessar um rio
conforme a figura. A corda foi amarrada em cada uma das extremidades de
FR = 0 ⇒  r modo a ficar θ θ
 ∑ Fy = 0 aproximadamente
horizontal se ninguém
estiver pendurado nela.
Estática do Corpo Extenso Quando pessoa atinge o
A estática do corpo extenso trata do equilíbrio estático dos corpos meio da travessia a
extensos, isto é, dos corpos que não podemos desprezar suas dimensões na tensão na corda é, em
análise do fenômeno. módulo, igual ao seu
A estática do corpo extenso é basicamente aplicar a condição de peso. Qual será o
equilíbrio de translação e a condição de equilíbrio de rotação, já que os corpos ângulo θ, em graus,
extensos podem viver movimento de rotação e/ou translação: nesta situação?
A condição de translação para o corpo extenso a mesma condição
aplicada para a partícula.
SE UM CORPO EXTENSO ESTÁ EM EQUILÍBRIO ESTÁTICO A FORÇA 163. (COVEST) Duas partículas idênticas, de massa m = 0,9kg cada, estão
RESULTANTE QUE ATUA SOBRE O MESMO É NULA. em repouso. Suspensas pro fios inextensíveis como mostrado na figura
O que adotando um sistema de referência da mesma forma descrita abaixo. Os fios A e B estão presos no teto e fazem ângulos de 45º com a
para as partículas leva-nos à mesma equação: vertical. A tração no fio horizontal C vale:
(a) 2,0N
SE UM CORPO EXTENSO ESTÁ EM EQUILÍBRIO ESTÁTICO: (b) 4,0N
r A
45º B
r  ∑ Fx = 0 (c) 6,0N 45º
(d)
FR = 0 ⇒  r (e)
8,0N C

 ∑
9,0N m m
Fy = 0
A condição de equilíbrio de rotação pode ser enunciada da seguinte 164. (COVEST) A figura mostra um peso de 44 N suspenso no ponto P de
forma: uma corda. Os trechos
AP e BP da corda
SE UM CORPO EXTENSO ESTÁ EM EQUILÍBRIO ESTÁTICO O
formam um ângulo de 60o B
TORQUE RESULTANTE QUE ATUA SOBRE O MESMO É NULO.
r 90o, e o ângulo entre BP
MR = 0 ⇒ ∑M = 0 e o teto é igual a 60o.
Qual é o valor, em
Na equação acima para facilitar o cálculo do torque calculamos seu newtons, da tração no A
módulo e adotamos a seguinte convenção para efetuar o somatório: trecho AP da corda?
• Se a força que provoca o torque tende a rotar o corpo no
sentido anti-horário o torque é positivo;
• Se a força que provoca o torque tende a rotar o corpo no P
sentido horário o torque é negativo.
165. (COVEST) A figura abaixo mostra um dispositivo constituído de um
Estabilidade do Equilíbrio suporte sobre o qual uma trave é apoiada. Na extremidade A, é suspenso
Podemos classificar o equilíbrio estático como sendo: um objeto, de massa 95 kg, enquanto se aplica uma força vertical F na
• Equilíbrio indiferente ou neutro  É aquele que ocorre quando extremidade B, de modo a equilibrar o objeto. Desprezando o peso da trave,
forças e/ou torques não existiram após a perturbação do equilíbrio no em relação ao peso do objeto, calcule o módulo da força F necessária para
sentido de restaurar o afastar o corpo da posição de equilíbrio. equilibrar o objeto, em N.
• Equilíbrio instável  É aquele que ocorre quando forças e/ou 0,5 m 5m
torques provocadas por um pequeno deslocamento do corpo em
relação a posição de equilíbrio provocam um afastamento ainda maior
A B
desta posição de equilíbrio. Isto é, se uma força e/ou torque deslocar o trave
corpo da posição de equilíbrio retira-o desta posição o afastando mais
ainda da antiga posição de equilíbrio.
• Equilíbrio estável  É aquele que ocorre quando forças e/ou torques
provocadas por um pequeno deslocamento do corpo atuam de modo
que fazem o corpo retornar à posição de equilíbrio. Isto é, o corpo é
suporte
perturbado da posição de equilíbrio mais retorna a esta após a
perturbação. 166. (COVEST) Uma barra horizontal de massa desprezível possui uma de
Em resumo podemos dizer que: se um sistema for ligeiramente suas extremidades articulada em uma parede vertical.
perturbado na sua posição de equilíbrio, o equilíbrio será estável se o sistema A outra extremidade está presa à
retornar espontaneamente para a sua posição de equilíbrio original, será instável parede por um fio que faz um ângulo fio
se o sistema se afastar espontaneamente da posição de equilíbrio original e será de 45º com a horizontal e possui um
indiferente (ou neutro) se nem forças e/ou torques atuarem para modificar a corpo de 55 N pendurado. Qual o 45º
posição perturbada do sistema. módulo da força normal à parede,
em newtons, que a articulação
exerce sobre a barra?

AUTORIA – PROF. MARCELO CORREIA 21 E-mail: marcelo.correia.fisica@bol.com.br


FÍSICA – MECÂNICA
167. (COVEST) A figura mostra uma barra homogênea, de comprimento FLUIDOSTÁTICA (HIDROSTÁTICA)
L=1,0m, presa ao teto nos pontos A e B por molas ideais, iguais de Vamos agora discutir a ultima parte da estática. A Fluidostática,
×102 N/m. A que distância do centro da barra, em
constante elástica k=1,0× comumente chamada de Hidrostática, estuda os fluidos que estão em equilíbrio
centímetros, deve ser pendurado um jarro de massa m=2,0 kg, de modo estático, isto é, que estão em repouso.
que a barra permaneça na horizontal? B
Densidade e Massa Específica
k1 = k2 = k A A densidade de um corpo será representada pela letra grega ρ (rô) e é
definida como sendo a razão entre a massa “m” do corpo e o seu volume “V”.
h = 0,1 m
Já a massa específica de uma substância representaremos pela letra grega µ
(mi) e é definida como a massa “m” de uma determinada porção da substância e
centro k2 o volume “V” ocupado por esta porção da substância.
k1
Observe que ambas as grandezas: densidade e massa específica são
razões da massa pelo volume. A diferença entre elas é que a densidade é para
corpos e a massa específica e par a substância. Por exemplo: Um bloco de ferro
m é um corpo e não precisa necessariamente ter densidade igual a massa
específica do ferro, isto só ocorre quando o corpo for homogêneo e maciço o que
168. (COVEST) A escada AB está apoiada numa parede sem atrito, no
será considerado na maioria dos casos, principalmente no nosso caso que
ponto B, e encontra-se na B estamos estando fluidos. Assim, se o corpo for homogêneo e maciço a densidade
iminência de escorregar. O e massa específica se tornam iguais. Matematicamente temos:
coeficiente de atrito estático entre a
escada e o piso é 0,25. Se a m m
distância de A até o ponto O é Densidade  ρ= Massa Específica  µ=
igual a 45 cm, qual a distância de V V
B até O, em centímetros? A unidade de densidade ou massa específica no SI é o: kg/m3. No
entanto existem outras unidades que são largamente usadas tais como: g/cm3;
O A kg/l. Abaixo colocamos uma equivalência que é freqüentemente utilizada para
facilitar na hora que precisarmos efetuar conversões de unidades:
1kg/m3 = 10–3 g/cm3
169. (COVEST) Deseja-se saber a massa de uma régua de 1,0 m de
Pressão
comprimento e dispõe-se de um pequeno corpo de 9,0 g. Realiza-se o
Como os fluidos não suportam tensões de cisalhamento (tangenciais)
experimento mostrado abaixo. Apóia-se a régua, na iminência de cair, sobre
os esforços que são exercidos sobre os fluidos ou os esforços que os fluidos
a borda de uma mesa horizontal, com o corpo na extremidade da régua (ver
exercem sobre os corpos devem ser medidos através da grandeza física escalar
figura). O ponto P coincide com a marcação 45 cm e alinha-se com a borda
que recebe o nome de pressão e é definida como sendo a razão entre o módulo
da mesa. O ponto Q indica o ponto médio da régua e o pequeno corpo
da força exercida e a área que a força atua. Sendo assim temos:
coincide com a marcação 0,0 cm. Calcule a massa da régua, em g.
régua Q P corpo F
p=
A
Observe que representamos a pressão por p (minúsculo) afim de não
confundirmos em algum problema pressão com peso (representado por P
(maiúsculo). A unidade de pressão no SI é o N/m2 (Newton por metro quadrado),
que recebe o nome especial de Pa (pascal), isto é, 1N/m2 = 1Pa. No entanto
existem outras unidades de pressão que são largamente usadas tais como: atm
170. (COVEST) A gangorra da figura abaixo está equilibrada em torno do (atmosfera), cmHg (centímetro de mercúrio). Abaixo mostramos a equivalência
ponto C por efeito das massas mA = 20kg e mB = 40kg. Indique o entre as unidades de pressão mais usadas:
comprimento total AB, em metros, supondo que AC = 6,0m. Despreze a facilitar na hora que precisarmos efetuar conversões de unidades:
massa da gangorra. A C B 105 Pa = 1atm = 76cmHg = 760mmHg
(a) 7,0
(b) 7,5 Teorema de Stevin
(c) 8,0 Podemos enunciar o teorema de Stevin da seguinte forma:
mA mB A DIFERENÇA DE PRESSÃO ENTRE DOIS PONTOS DE UM FLUIDO
(d) 8,5
HOMOGÊNEO EM EQUILÍBRIO É DADA PELA PRESSÃO HIDROSTÁTICA DA
(e) 9,0
COLUNA DE FLUIDO ENTRE ESTES DOIS PONTOS.
171. (COVEST) Uma tábua uniforme de peso igual a 48N e 3,6m de Matematicamente temos:
comprimento repousa horizontalmente sobre dois cavaletes, conforme a A
figura. Qual a força normal, em N, exercida sobre a tabula no ponto P? pB = p A + ρ ⋅ g ⋅h
123 h
2,4m 1,2m pressão hidrostática
B
g  aceleração da gravidade
ρ  densidade do fluido
P Se o fluido tem a superfície livre, isto é, o ponto A é a superfície do
fluido nesta atua a pressão atmosférica do local, representando a pressão
atmosférica por p0 o teorema de Stevin para o cálculo da pressão p num ponto a
uma profundidade h da superfície do fluido pe dado por:
p = p0 + ρ ⋅ g ⋅h
AUTORIA – PROF. MARCELO CORREIA 22 E-mail: marcelo.correia.fisica@bol.com.br
FÍSICA – MECÂNICA
Teorema de Pascal 174. (COVEST) Um tubo em U, aberto em ambas as extremidades e de
Podemos enunciar o teorema de Stevin da seguinte forma: seção reta uniforme, contém uma certa quantidade de água. Adiciona-se
UMA VARIAÇÃO DE PRESSÃO PROMOVIDA NUM PONTO QUALQUER DE 500mL de um líquido imiscível, de densidade ρ=0,8g/cm3, no ramo da
UM FLUIDO EM EQUILÍBRIO, SE TRANSMITE INTEGRALMENTE PARA esquerda. Qual o peso do êmbolo, em newtons, que deve ser colocado no
TODOS OS PONTOS DO FLUIDO. ramo da direita, para que os níveis de água nos dois ramos sejam iguais?
A principal aplicação do teorema de pascal é o que chamamos de Despreze o atrito do êmbolo com as paredes do tubo.
prensa hidráulica. A prensa hidráulica consiste dois recipientes verticais
(comumente cilíndricos) de secções retas distintas interligadas por um tubo, no
interior do dispositivo é colocado um fluido que sustenta dois êmbolos móveis.
Abaixo mostramos uma figura esquemática da prensa hidráulica e a equação líquido
deduzida a partir do teorema de pascal: êmbolo
F2
F1 água
A2
F1 F2 A1 175. (COVEST) A figura mostra dois recipientes, cujas bases têm áreas que
= satisfazem à relação A1=3A2. Coloca-se 33 litros de água nestes
A1 A2 recipientes, até atingir o nível h. Determine a força exercida pela água
sobre a base do recipiente 2, em kgf. Despreze o efeito da pressão
atmosférica.
recipiente 1 recipiente 2
Teorema de Arquimedes ou Teorema do Empuxo
Podemos enunciar o teorema de Stevin da seguinte forma:
TODO CORPO TOTAL OU PARCIALMENTE IMERSO NUM FLUIDO EM
EQUILÍBRIO, FICA SUJEITO A AÇÃO DE UMA FORÇA VERTICAL E
APONTANDO PARA CIMA QUE RECEBE O NOME DE EMPUXO E TEM
MÓDULO IGUAL AO PESO DE FLUIDO DESLOCADO PELO CORPO. h
Matematicamente podemos escrever:
A1 A2
E = Pfluido deslocado
Depois de aplicar a definição de peso e densidade podemos chegar a 176. (COVEST) É impossível para uma pessoa respirar se a diferença de
seguinte expressão equivalente à anterior: pressão entre o meio externo e o ar dentro dos pulmões for maior do que
0,05atm. Calcule a profundidade máxima, h, dentro d’água, em cm, na qual
E = ρ f ⋅ Vfd ⋅ g um mergulhador pode respirar por meio de um tubo, cuja extremidade
Onde: ρf  densidade do fluido; Vfd  volume de fluido deslocado e superior é mantida fora da água.
g  aceleração da gravidade

BRINQUEDINHO DE VESTIBULANDO!!!!!
172. (COVEST) Uma mola ideal de comprimento L=65cm está presa no
fundo de uma piscina que está sendo cheia. Um cubo de isopor de aresta
a=10cm e massa desprezível é preso na extremidade superior da mola. O
cubo fica totalmente coberto no instante em que o nível da água atinge a
altura H=1,0m em relação ao fundo da piscina. Calcule a constante elástica 177. (COVEST) Duas esferas de mesmo raio e massas MA=0,5 kg e
da mola, em N/m. MB=0,3 kg, estão presas por
um fio fino, inextensível e de
massa desprezível, conforme MB
mostra a figura. As esferas
a encontram-se em repouso,
H imersas em um líquido.
Determine o empuxo exercido
L pelo líquido sobre cada
esfera.
178. (UPE) A famosa experiência de Torricelli foi realizada com o mercúrio,
173. (COVEST) Um bloco homogêneo e impermeável, de densidade porque:
ρ=0,25g/cm3, está em repouso, imerso em um tanque completamente cheio (a) Se fosse feita com a água, que apresenta densidade muito inferior à do
de água e vedado, como mostrado na figura a seguir. Calcule a razão entre mercúrio, a altura seria imperceptível.
os módulos da força que o bloco exerce na tampa superior do tanque e do (b) Se fosse feita com um líquido mais denso que o mercúrio, o tubo de vidro
peso do bloco. deveria ter maior comprimento.
(c) O mercúrio é o único metal em estado líquido, na temperatura ambiente.
tampa
(d) O mercúrio, sendo um metal líquido, é bom condutor de calor.
(e) Se fosse feita com a água, com densidade muito menor que a do mercúrio,
o tubo de vidro deveria ter comprimento maior que 10 m.
água

AUTORIA – PROF. MARCELO CORREIA 23 E-mail: marcelo.correia.fisica@bol.com.br

Potrebbero piacerti anche